Download PPS Exam March 20, 2002

Survey
yes no Was this document useful for you?
   Thank you for your participation!

* Your assessment is very important for improving the workof artificial intelligence, which forms the content of this project

Document related concepts

Dental emergency wikipedia , lookup

Infection control wikipedia , lookup

Dysprosody wikipedia , lookup

Canine parvovirus wikipedia , lookup

Prenatal testing wikipedia , lookup

List of medical mnemonics wikipedia , lookup

Transcript
a.
b.
PPS Exam March 20, 2002
Recalled Questions
1.
IMCI: Treatment for Shigella
a. Ampicillin for 5 days
2.
IMCI: case of a child with pneumonia
a. send home
b. treat for 5 days
16. 5 year old without previous vaccination on 1st
visit give:
a. DPT1, OPV1, Hib1
b. DPT1, HepB1
c. DPT1, OPV1, Hib1, MMR
3. IMCI: infant with persistent diarrhea but no
dehydration
a. continue breastfeeding
b. do not give vitamin A
c. is still dehydrated
4.
IMCI: case of an ill infant not feeding
a. small feedings of soft, preferred foods and
continue breastfeeding
b. go to hospital
5.
Treatment for allergic rhinitis
a. topical corticosteroids
6.
No longer used in neonatal resuscitation:
a. albumin containing solutions
b. intraosseous access
c. NaHCO3 (causes burns)
7.
Use of vaccines with thiomerosal when no
other thiomerosal-free vaccine is available:
a. use
b. do not use
c. use only for Hep B and Pertussis
8.
Med rep gives you a free trip to Hongkong for
prescribing their milk formula for a year:
a. accept
b. do no accept
c. report
d. ask your department chairman
9.
15 year old with metastatic lung Ca asks for
morphine for pain-relief:
a. give to relieve pain
b. do not give
c. start aggressive chemotherapy
10. Fellow takes a look at the patient of a
consultant who is not available. What will you
do?
a. tell the parents
b. report the incident
c. talk to the Fellow
11. Routine
measurement
of
the
circumference is taken up to this age:
a. 2
b. 3
c. 4
head
12. U/L ratio is equal to 1 at this age:
a. 3
b. 5
c. 7
13. Can say other words beside Mama and Papa:
a. 6 mos
b. 8 mos
c. 10 mos
d. 12 mos
14. Toilet training:
a. 24-48 mos
b. 48-60 mos
15. EPI vaccination at 1 year old:
DPT3, OPV3, HepB3, Measles
DPT3, OPV3, HepB3
17. Asthma patient with PEFR >30, daily
symptoms for a week, night symptoms once a
week:
a. moderate persistent
b. severe persistent
18. Findings in restrictive lung disease:
a. increase RR (plus other choices I can’t
remember)
b. decrease RR
19. Premature formulas should be discontinued at
34-36 weeks due to:
a. hypercalcemia
b. hypernatremia
c. high ash content which may damage
kidneys
20. Freshwater drowning:
a. hyperosmolality
b. hypoosmolality
c. hyponatremia and hemodilution
d. hypernatremia
21. Preventive pediatrics – monthly visits during
the 1st year. On the 2nd year, well-baby visits
every:
a. 12 months
b. 3 months
c. 4 months
d. 6 months
22. 17 month weighing 5 kg. Weight for age based
on the Waterlow Classification:
a. normal
b. mild
c. moderate
d. severe
23. IMCI moderate dehydration (Plan B) in a 7 kg
child
a. 300 ml
b. 500 ml
c. 700 ml
24. 2 year old with severe dehydration. Fluid to be
given over 30 minutes:
a. 30 ml/kg
b. 70 ml/kg
25. Dengue in shock. What to give after LRS
bolus?
a. D5 IMB
b. D5 NM
c. D5 0.3 NaCl
d. Colloid
26. 3 wk old 1.900 kg fed 30 ml every 3 hours with
abdominal distention and vomiting.
a. CBC
b. Urinalysis
c. Abdominal x-ray
d. Ultrasound
27. An
example
of
pathologic
indirect
hyperbilirubinemia caused by increased
bilirubin production:
a. prematurity
b.
c.
d.
sepsis
inborn errors
ABO incompatibility
28. Type of Polio virus in the 3 cases identified in
the Philippines:
a. I
b. II
c. III
d. IV
29. Hypersensitivity reaction wherein an antigen is
presented to a sensitized cell:
a. I
b. II
c. III
d. IV
30. Maneuver to
intubation:
a. Sellick’s
prevent
aspiration
during
31. X-linked recessive trait:
a. all daughters of affected males are carriers
32. Perinatal cause of mental retardation:
a. hypoxic-ischemic injury
b. chromosomal
33. Equinus gait / toe walking until:
a. 3 years old
34. Single umbilical artery is most commonly
associated with:
a. trisomy
b. Potter syndrome
35. Most common with good prognosis:
a. cerebellar astrocytoma
36. Congenital scoliosis is most commonly
associated with malformations in this organ
system:
a. genitourinary
b. cardiac
c. pulmonary
37. Clinical triad of short neck, low hairline, and
restricted neck motion with multiple coalitions:
a. Klippel-Feil
b. Sprengel
c. Noonan’s
(?) Cornelia de Lange
42. Management of caustic ingestion except:
a. endoscopy should not be done in 24-48
hours
b. give milk or water
c. no need for lavage
43. Antidote for acetaminophen overdose
a. NAC
44. Which blocks acetylcholine receptors
a. organophosphates
45. Safe for breastfeeding mothers:
a. paracetamol
46. True of kerosene ingestion except:
a. aspiration pneumonia should be treated
with antibiotics
b. no need for lavage
c. (forgot the other choices. Sorry)
47. Increased risk of kernicterus by displacing
albumin:
a. sulfonamide
b. chloramphenicol
c. tetracycline
48. Theophylline decreases
increasing levels of:
a. terbutaline
b. rifampicin
c. phenobarbital
d. erythromycin
clearance
thereby
49. Cyanotic neonate with findings of LVH:
a. TOGA
b. TOF
50. True of CHF in neonates:
a. digoxin dosage is the same as in older
children
b. edema not limited to lower extremities
51. Management of cardiogenic shock except:
a. dopamine to maintain cardiac output
b. epinephrine to maintain heart rate
c. digitalis
d. treat cause
52. Seizures lasting for 10-20 seconds but patient
still speaks without loss of consciousness:
a. simple partial seizure
b. complex
c. absence
38. Not an absolute contraindication to sports:
a. murmur
b. seizure in swimming activities
c. hyperpyrexia
53. 3 year old with seizure for 45 seconds with
normal PE except T 39.5C
a. febrile seizure
39. Immediate management of sports injury except:
a. ice
b. compression
c. NSAID
d. Massage ligament
54. Girl with SMR Tanner 5
amenorrhea and midline mass:
a. ovarian tumor
b. imperforate hymen
c. uterine tumor
40. Oligohydramnios associated with:
a. clubfoot
b. spadelike hands
c. flattened nasal bridge
d. all
55. 15 year old with multiple tender bilateral breast
masses advised by a surgeon to be premalignant. What would you do?
a. give
oral
contraceptive
pills
with
progesterone
b. do aspiration biopsy
c. do excision biopsy
d. agree with surgeon and remove masses
41. Increased ICP
a. lead
b. salicylate
c. INH
d. Mercury
with
primary
56. Child given vitamins A,D, and C for several
months with anorexia, irritability, and tender
bone swelling:
a. hypervitaminosis A
b. hypervitaminosis C
c. hypervitaminosis B
d. Caffey’s disease
57. Most common anatomic abnormality in patient
with OSAS:
a. adenoidal hypertrophy
b. obesity
58. Pre-eclamptic mother admitted for several days
gives birth to an apneic infant. Check for:
a. glucose
b. Ca
c. Mg
d. All
71. Leading cause of death in the adolescent age
group:
a. suicide
b. motor vehicle
c. injury
72. Use of alcohol increases effects of which
abused substance:
a. amphetamines
b. cocaine
c. marijuana
73. Adolescent caught stealing twice and driving
without a license:
a. juvenile delinquency
b. oppositional defiant disorder
59. Vitamin K deficiency:
a. bleeding time
b. clotting time
c. prothrombin Factor V
d. one step prothrombin
74. Child with almost monthly skin abscess even
with frequent hexachlorophene washes.
a. T cell deficiency
b. B cell deficiency
c. NK cell deficiency
60. Requirement for the diagnosis of kwashiorkor:
a. Edema
75. After intake of Amoxicillin, noted skin erythema,
(+) Nikolsky, (-) perioral crusting.
a. Ritter
b. TEN
c. epidermolysis bullosa
d. erythema multiforme
61. Necessary to prevent neural tube defects in
newborns:
a. folate
62. Newborn screening which detected an
abnormality not thought to be common in the
Philippines:
a. galactosemia
b. phenylketonuria
c. homocystinuria
76. In-vitro test for IgE
a. RAST
77. No renal abnormalities
a. SLE
b. PAN
c. dermatomyositis
63. ABG values interpretation
78. case of bronchiolitis and management
64. Most common cause of diabetes insipidus:
a. trauma
b. craniopharyngioma
79. case of epiglotittis and management
80. case of AGN
65. Anaerobic bacteria
a. meningitis
b. abscess
c. ventriculitis
81. case of SLE
diagnostics)
(including
statistics
and
82. case of HSP
66. Not in ARDS:
a. hypoxemia
b. diffuse pulmonary infiltrates
c. normal pulmonary artery pressure
d. abnormal cardiac function
67. Test for hemophilia
a. PT
b. PTT
c. Factor assay
68. Milk let down signifies:
a. successful nursing
b. unsuccessful nursing
c. maternal fatigue
d. maternal anxiety
69. Test for strabismus in an uncooperative child:
a. cover test
70. Nadir of glucose in IDM:
a. 1-3 hours
b. 30 minutes
c. 4-6 hours
83. case of JRA (?)
84. True of Kawasaki except
a. treatment with IVIG and steroids
85. A child takes the Phenobarbital tablets used by
the mother for a sibling with seizure disorder
anytime before or after school. This behavior
is:
a. serious
b. very serious
c. no effect
86. Baby fat is burned at this age:
a. 1 year
b. 2 years
c. 3 years
d. 4 years
87. Taste discrimination at what age for infants
3 months
88. Case of toxic shock syndrome. Symptoms are
due to the ff. except:
a. lipotechoic-peptidoglycan complex
b.
c.
d.
IL
TNF
endotoxin
103. case of meningitis – N. meningitidis
104. case of croup – with stridor, etc.
89. Case of meningococcemia. Symptoms are due
to:
a. lipotechoic-peptidoglycan complex
b. IL
c. TNF
d. endotoxin
90. Does not cross the blood-brain-barrier
a. cotrimoxazole
b. chloramphenicol
c. INH
d. rifampicin
91. Neonate placed on mechanical ventilator.
Parents want to discontinue treatment.
a. consult bioethics committee
92. Congenital syphilis is transmitted during:
a. 1st trimester
b. 2nd trimester
c. 3rd trimester
d. any
105. Child with FUO, history of pica and has a dog:
a. lead
b. Toxocara
106. Prophylaxis for endocarditis prior to dental
extraction.
107. Premature neonate with abdominal flank mass,
hematuria.
a. renal vein thrombosis
108. Poor prognostic indicator in ALL:
a. young age at onset
b. hyperploidy
with
more
than
50
chromosomes
c. chromosomal translocation
d. rearrangement of TEL/AMLI genes in Bprogenitor ALL
109. case of tetanus
93. Iron deficiency anemia
a. 3-6 months
b. 9-24 months
110. diagnosis of rheumatic fever
94. Not part of management of recurrent AOM
a. adenoidectomy
b. myringotomy
112. Child with mother diagnosed to have TB. PPD
is 2 mm. CXR negative. No signs and
symptoms.
a. treat with INH then repeat PPD after 3
months
95. Labial adhesions need not be
aggressively because:
a. systemic steroids are effective
b. resolve by adolescence
111. treatment of extrapulmonary TB
treated
96. Case of Wiskott Aldrich syndrome
113. Flouride concentration in water to prevent
dental caries:
a. 1.0 –1.5 ppm
b. 2.0 – 2.5 ppm
c. 3 – 3.5 ppm
97. Language development is important during this
period for later school success:
a. infancy
b. pre-school
c. school age
98. Typhoid fever resistant to the usual antibiotics
can be treated with:
a. ciprofloxacin
b. cotrimoxazole
c. ceftriaxone
99. Not true about the ff. drug effects on the fetus:
a. valproic acid –
b. phenytoin –
c. retinoin –
d. alcohol –
100. Not true about hypertension:
a. Most common cause in 2-4 years old is
renal
b. Drug therapy can be used in the 1 st stage
of management
101. Not true about drug absorption:
a. Systemic absorption of a drug given
percutaneously is absorbed 3x more in
children.
b. Drugs given IV are 100% bioavailable.
c. Only unbound, protein-free drugs are
filtered freely in the kidney.
102. Amount of glucose in WHO ORS:
a. 10 g/dl
c. 30 g/dl
b. 20 g/dl
d. 40 g/dl
(?) 111 g/dl
celineblancas-evidente
slmc / march 2002
c.
d.
PPS Exam March 20, 2002
Recalled Questions
3.
IMCI: Treatment for Shigella
b. Ampicillin for 5 days
4.
IMCI: case of a child with pneumonia
c. send home
d. treat for 5 days
126. 5 year old without previous vaccination on 1 st
visit give:
d. DPT1, OPV1, Hib1
e. DPT1, HepB1
f. DPT1, OPV1, Hib1, MMR
3. IMCI: infant with persistent diarrhea but no
dehydration
d. continue breastfeeding
e. do not give vitamin A
f. is still dehydrated
114. IMCI: case of an ill infant not feeding
c. small feedings of soft, preferred foods and
continue breastfeeding
d. go to hospital
115. Treatment for allergic rhinitis
b. topical corticosteroids
116. No longer used in neonatal resuscitation:
d. albumin containing solutions
e. intraosseous access
f. NaHCO3 (causes burns)
117. Use of vaccines with thiomerosal when no
other thiomerosal-free vaccine is available:
d. use
e. do not use
f. use only for Hep B and Pertussis
118. Med rep gives you a free trip to Hongkong for
prescribing their milk formula for a year:
e. accept
f. do no accept
g. report
h. ask your department chairman
119. 15 year old with metastatic lung Ca asks for
morphine for pain-relief:
d. give to relieve pain
e. do not give
f. start aggressive chemotherapy
120. Fellow takes a look at the patient of a
consultant who is not available. What will you
do?
d. tell the parents
e. report the incident
f. talk to the Fellow
121. Routine
measurement
of
the
circumference is taken up to this age:
d. 2
e. 3
f. 4
head
122. U/L ratio is equal to 1 at this age:
d. 3
e. 5
f. 7
123. Can say other words beside Mama and Papa:
e. 6 mos
f. 8 mos
g. 10 mos
h. 12 mos
124. Toilet training:
c. 24-48 mos
d. 48-60 mos
125. EPI vaccination at 1 year old:
DPT3, OPV3, HepB3, Measles
DPT3, OPV3, HepB3
127. Asthma patient with PEFR >30, daily
symptoms for a week, night symptoms once a
week:
c. moderate persistent
d. severe persistent
128. Findings in restrictive lung disease:
c. increase RR (plus other choices I can’t
remember)
d. decrease RR
129. Premature formulas should be discontinued at
34-36 weeks due to:
d. hypercalcemia
e. hypernatremia
f. high ash content which may damage
kidneys
130. Freshwater drowning:
e. hyperosmolality
f. hypoosmolality
g. hyponatremia and hemodilution
h. hypernatremia
131. Preventive pediatrics – monthly visits during
the 1st year. On the 2nd year, well-baby visits
every:
e. 12 months
f. 3 months
g. 4 months
h. 6 months
132. 17 month weighing 5 kg. Weight for age based
on the Waterlow Classification:
e. normal
f. mild
g. moderate
h. severe
133. IMCI moderate dehydration (Plan B) in a 7 kg
child
d. 300 ml
e. 500 ml
f. 700 ml
134. 2 year old with severe dehydration. Fluid to be
given over 30 minutes:
c. 30 ml/kg
d. 70 ml/kg
135. Dengue in shock. What to give after LRS
bolus?
e. D5 IMB
f. D5 NM
g. D5 0.3 NaCl
h. Colloid
136. 3 wk old 1.900 kg fed 30 ml every 3 hours with
abdominal distention and vomiting.
e. CBC
f. Urinalysis
g. Abdominal x-ray
h. Ultrasound
137. An
example
of
pathologic
indirect
hyperbilirubinemia caused by increased
bilirubin production:
e. prematurity
f.
g.
h.
sepsis
inborn errors
ABO incompatibility
138. Type of Polio virus in the 3 cases identified in
the Philippines:
e. I
f. II
g. III
h. IV
139. Hypersensitivity reaction wherein an antigen is
presented to a sensitized cell:
e. I
f. II
g. III
h. IV
140. Maneuver to
intubation:
b. Sellick’s
prevent
aspiration
during
141. X-linked recessive trait:
b. all daughters of affected males are carriers
142. Perinatal cause of mental retardation:
c. hypoxic-ischemic injury
d. chromosomal
143. Equinus gait / toe walking until:
b. 3 years old
144. Single umbilical artery is most commonly
associated with:
c. trisomy
d. Potter syndrome
145. Most common with good prognosis:
b. cerebellar astrocytoma
146. Congenital scoliosis is most commonly
associated with malformations in this organ
system:
d. genitourinary
e. cardiac
f. pulmonary
147. Clinical triad of short neck, low hairline, and
restricted neck motion with multiple coalitions:
d. Klippel-Feil
e. Sprengel
f. Noonan’s
(?) Cornelia de Lange
152. Management of caustic ingestion except:
d. endoscopy should not be done in 24-48
hours
e. give milk or water
f. no need for lavage
153. Antidote for acetaminophen overdose
b. NAC
154. Which blocks acetylcholine receptors
b. organophosphates
155. Safe for breastfeeding mothers:
b. paracetamol
156. True of kerosene ingestion except:
d. aspiration pneumonia should be treated
with antibiotics
e. no need for lavage
f. (forgot the other choices. Sorry)
157. Increased risk of kernicterus by displacing
albumin:
d. sulfonamide
e. chloramphenicol
f. tetracycline
158. Theophylline decreases
increasing levels of:
e. terbutaline
f. rifampicin
g. phenobarbital
h. erythromycin
clearance
thereby
159. Cyanotic neonate with findings of LVH:
c. TOGA
d. TOF
160. True of CHF in neonates:
c. digoxin dosage is the same as in older
children
d. edema not limited to lower extremities
161. Management of cardiogenic shock except:
e. dopamine to maintain cardiac output
f. epinephrine to maintain heart rate
g. digitalis
h. treat cause
162. Seizures lasting for 10-20 seconds but patient
still speaks without loss of consciousness:
d. simple partial seizure
e. complex
f. absence
148. Not an absolute contraindication to sports:
d. murmur
e. seizure in swimming activities
f. hyperpyrexia
163. 3 year old with seizure for 45 seconds with
normal PE except T 39.5C
b. febrile seizure
149. Immediate management of sports injury except:
e. ice
f. compression
g. NSAID
h. Massage ligament
164. Girl with SMR Tanner 5
amenorrhea and midline mass:
d. ovarian tumor
e. imperforate hymen
f. uterine tumor
150. Oligohydramnios associated with:
e. clubfoot
f. spadelike hands
g. flattened nasal bridge
h. all
165. 15 year old with multiple tender bilateral breast
masses advised by a surgeon to be premalignant. What would you do?
e. give
oral
contraceptive
pills
with
progesterone
f. do aspiration biopsy
g. do excision biopsy
h. agree with surgeon and remove masses
151. Increased ICP
e. lead
f. salicylate
g. INH
h. Mercury
with
primary
166. Child given vitamins A,D, and C for several
months with anorexia, irritability, and tender
bone swelling:
e. hypervitaminosis A
f. hypervitaminosis C
g. hypervitaminosis B
h. Caffey’s disease
167. Most common anatomic abnormality in patient
with OSAS:
c. adenoidal hypertrophy
d. obesity
168. Pre-eclamptic mother admitted for several days
gives birth to an apneic infant. Check for:
e. glucose
f. Ca
g. Mg
h. All
181. Leading cause of death in the adolescent age
group:
d. suicide
e. motor vehicle
f. injury
182. Use of alcohol increases effects of which
abused substance:
d. amphetamines
e. cocaine
f. marijuana
183. Adolescent caught stealing twice and driving
without a license:
c. juvenile delinquency
d. oppositional defiant disorder
169. Vitamin K deficiency:
e. bleeding time
f. clotting time
g. prothrombin Factor V
h. one step prothrombin
184. Child with almost monthly skin abscess even
with frequent hexachlorophene washes.
d. T cell deficiency
e. B cell deficiency
f. NK cell deficiency
170. Requirement for the diagnosis of kwashiorkor:
b. Edema
185. After intake of Amoxicillin, noted skin erythema,
(+) Nikolsky, (-) perioral crusting.
e. Ritter
f. TEN
g. epidermolysis bullosa
h. erythema multiforme
171. Necessary to prevent neural tube defects in
newborns:
b. folate
172. Newborn screening which detected an
abnormality not thought to be common in the
Philippines:
d. galactosemia
e. phenylketonuria
f. homocystinuria
186. In-vitro test for IgE
b. RAST
187. No renal abnormalities
d. SLE
e. PAN
f. dermatomyositis
173. ABG values interpretation
188. case of bronchiolitis and management
174. Most common cause of diabetes insipidus:
c. trauma
d. craniopharyngioma
189. case of epiglotittis and management
190. case of AGN
175. Anaerobic bacteria
d. meningitis
e. abscess
f. ventriculitis
191. case of SLE
diagnostics)
(including
statistics
and
192. case of HSP
176. Not in ARDS:
e. hypoxemia
f. diffuse pulmonary infiltrates
g. normal pulmonary artery pressure
h. abnormal cardiac function
177. Test for hemophilia
d. PT
e. PTT
f. Factor assay
178. Milk let down signifies:
e. successful nursing
f. unsuccessful nursing
g. maternal fatigue
h. maternal anxiety
179. Test for strabismus in an uncooperative child:
b. cover test
180. Nadir of glucose in IDM:
d. 1-3 hours
e. 30 minutes
f. 4-6 hours
193. case of JRA (?)
194. True of Kawasaki except
b. treatment with IVIG and steroids
195. A child takes the Phenobarbital tablets used by
the mother for a sibling with seizure disorder
anytime before or after school. This behavior
is:
d. serious
e. very serious
f. no effect
196. Baby fat is burned at this age:
e. 1 year
f. 2 years
g. 3 years
h. 4 years
197. Taste discrimination at what age for infants
3 months
198. Case of toxic shock syndrome. Symptoms are
due to the ff. except:
e. lipotechoic-peptidoglycan complex
f.
g.
h.
IL
TNF
endotoxin
213. case of meningitis – N. meningitidis
214. case of croup – with stridor, etc.
199. Case of meningococcemia. Symptoms are due
to:
e. lipotechoic-peptidoglycan complex
f. IL
g. TNF
h. endotoxin
200. Does not cross the blood-brain-barrier
e. cotrimoxazole
f. chloramphenicol
g. INH
h. rifampicin
201. Neonate placed on mechanical ventilator.
Parents want to discontinue treatment.
b. consult bioethics committee
202. Congenital syphilis is transmitted during:
e. 1st trimester
f. 2nd trimester
g. 3rd trimester
h. any
215. Child with FUO, history of pica and has a dog:
c. lead
d. Toxocara
216. Prophylaxis for endocarditis prior to dental
extraction.
217. Premature neonate with abdominal flank mass,
hematuria.
b. renal vein thrombosis
218. Poor prognostic indicator in ALL:
e. young age at onset
f. hyperploidy
with
more
than
50
chromosomes
g. chromosomal translocation
h. rearrangement of TEL/AMLI genes in Bprogenitor ALL
219. case of tetanus
203. Iron deficiency anemia
c. 3-6 months
d. 9-24 months
220. diagnosis of rheumatic fever
204. Not part of management of recurrent AOM
c. adenoidectomy
d. myringotomy
222. Child with mother diagnosed to have TB. PPD
is 2 mm. CXR negative. No signs and
symptoms.
b. treat with INH then repeat PPD after 3
months
205. Labial adhesions need not be
aggressively because:
c. systemic steroids are effective
d. resolve by adolescence
221. treatment of extrapulmonary TB
treated
206. Case of Wiskott Aldrich syndrome
223. Flouride concentration in water to prevent
dental caries:
d. 1.0 –1.5 ppm
e. 2.0 – 2.5 ppm
f. 3 – 3.5 ppm
207. Language development is important during this
period for later school success:
d. infancy
e. pre-school
f. school age
208. Typhoid fever resistant to the usual antibiotics
can be treated with:
d. ciprofloxacin
e. cotrimoxazole
f. ceftriaxone
209. Not true about the ff. drug effects on the fetus:
e. valproic acid –
f. phenytoin –
g. retinoin –
h. alcohol –
210. Not true about hypertension:
c. Most common cause in 2-4 years old is
renal
d. Drug therapy can be used in the 1 st stage
of management
211. Not true about drug absorption:
d. Systemic absorption of a drug given
percutaneously is absorbed 3x more in
children.
e. Drugs given IV are 100% bioavailable.
f. Only unbound, protein-free drugs are
filtered freely in the kidney.
212. Amount of glucose in WHO ORS:
c. 10 g/dl
c. 30 g/dl
d. 20 g/dl
d. 40 g/dl
(?) 111 g/dl
celineblancas-evidente
slmc / march 2002
1. Infant mortality rate, based from the
Philippine statistics, from 1994-1998:
46 per 1,000 livebirths (p.6t, del
Mundo, 4th ed.)
2. This is considered as the most common
nutritional disorder in the world:
Iron deficiency anemia (p.11, del
Mundo, 4th ed.)
3. Reduces the occurrence of neural tube
defects:
Folic Acid
4. Included in the EPI, EXCEPT:
Haemophilis Infuenza type B
vaccine (p.42t, del Mundo, 4th ed.)
5. True about vaccination with live virus
vaccine in pregnant women:
6. Most common site of accident in
preschoolers:
7. When talking to parents regarding the
health needs of their young adolescents, the
most frequent aspect discussed is:
8. Pre-exposure rabies vaccination schedule
that is economical:
9. A mother who was positive for VDRL gave
birth. The newborn is at risk of having
congenital syphilis in the following, EXCEPT:
(Pls. Check risk factors for congenital syphilis
p.905, Nelson)
10. A mother was found to be reactive for
HBsAg. At birth, which of the following should
be done:
Give Hep B vaccine only
Mother may breastfeed ?
11. True contraindications to vaccine:
12. Which of the following vaccines can be
given to an immunocompromised person,
considering that he is not severely
immunocompromised:
Rotavirus ?
BCG
MMR
OPV
13. Anticonvulsants produce blockade to Na
channels and increase the threshold for
calcium… Which of the following
anticonvulsant can produce a Reye-like
syndrome?
Valproic acid (p.1823, Nelson)
14. The relative size of the brain to total body
weight at birth is:
10% (p. 53, del Mundo, 4th ed.)
15. Which of the following structures carries
highly oxygenated blood during fetal
circulation?
Umbilical vein (p.930, del Mundo, 4th
ed.)
16. Cord blood fetal hemoglobin at birth is
how many % of the total blood volume?
80% (p. 55, del Mundo, 4th ed.)
17. A newborn was placed on his side. The
newborn was longitudinally transected by
a red colored lower part and a palecolored upper part. This phenomenon is:
Harlequin color change
18. The bacterial flora of the gut is
established during the first few hours after
birth. Breastfed infants have:
Lactobacillus bifidus (p.56, del
Mundo, 4th ed.)
19. Maintenance fluid is given with a higher
osmolality in adults as compared to
infants because:
The total body water is higher in
infants than in adults. (?)
20. A 5 year old female has short stature,
delayed dentition, enlarged protruding
tongue, dry, course skin, constipation,
history of prolonged jaundice:
Congenital Hypothyroidism
21. An eight year old has a weight of 15 kg.
and a height of 115 cm. Based on the
Waterlow classification for wasting and
stunting, this child has:
Severe wasting, mild stunting (p.
79, del Mundo, 4th ed.)
22. The height is 3x the birth length at:
13 years (p. 80, del Mundo, 4th ed. )
23. From the 6th to the 20th year, the head
circumference increases to ½ inch per
___ yrs.
5 years (p. 80, del Mundo, 4th ed.)
24. The crown-symphysis to symphisis-sole
measurements have a ratio of 1.7 at birth
and become 1 at ____ years.
10 years (p.83, del Mundo, 4th ed.)
25. Providing extraordinary means for a
known fatal ailment:
26. Filipino infants are able to hold the breast
or the bottle at about the:
Fourth month (p.101, del Mundo, 4th
ed.)
27. Writing movements follow a specific
pattern. The child’s style of writing is
quite set at ____ years and penmanship
remains such up to adulthood with minor
changes.
12 years (p. 101, dl Mundo, 4th ed.)
28. At 18 months, the infant has:
10-12 simple words (p. 101, del
Mundo, 4th ed.)
29. The IQ is classified as dull or borderline if
it is between:
70-79 (p.116, del Mundo, 4th ed.)
30. This macronutrient is not essential for
term infants in early life:
Nucleotides (p. 130, del Mundo, 4th
ed.)
31. A newborn who has persistent vomiting
and dehydration has:
Metabolic alkalosis
32. Bone age is determined by having an xray of the left:
wrist
33. In wiskott-aldrich, this is characterized by
thrombocytopenia and:
Atopic dermatitis (p. 604, Nelson,
16th ed.)
34. Organism associated with necrotizing
enterocolitis:
Clostridium perfringens (p. 512,
Nelson, 16th ed.)
35. Pyloric stenosis is a worldwide.. . This is
a risk factor and predisposes newborns to
pyloric stenosis:
Males and first born (p.1130,
Nelson, 16th ed.)
36. A child with abdominal pain, arthritis and
purpura on the buttock:
HSP
37. A child with abdominal pain, purpura on
the buttocks and lower extremities,
developed hematuria:
HSP nephropathy
38. A 5 year old presented at the ER with
fever, malaise, chills, headache,
arthralgia. BP: 70/40, with petechiae on
the chest. Most like he has:
Meningococcemia (p. 455, del
Mundo, 4th ed.)
39. A child with polyuria and polydipsia with
10% weight loss over the past few
months, presented at the ER with …..
urinalysis showed glucose (+++) and
ketones (+++). The most likely condition
is:
Type I diabetes mellitus
40. A child with polyuria and polydipsia
underwent a water deprivation test. After
the test, the SG increased from 1.001 to
1.025 and the volume of the urine
______. This child most likely has:
Psychogenic polydipsia (p.1195, del
Mundo, 4th ed.)
41. This is NOT likely an associated finding in
Kawasaki disease:
Thrombocytopenia
42. The most frequent site of coarctation of
the aorta is :
just below the origin of the
subclavian artery (p.1379, Nelson, 16th
ed.)
43. A substernal thrust indicates the presence
of:
Right ventricular enlargement (p.
1347, Nelson, 16th ed.)
44. Skimmed milk is not given to a 9 month
old because:
Its high protein and mineral content
in proportion to calories may cause
severe dehydration (p. 157, Nelson)
45. At 6 months a mother started weaning her
baby with cereals. However, the baby
would spit the cereals. What would you
advise?
Do not force the child to eat. (p.
157, del Mundo, 4th ed.)
46. The newborn is a homoiotherm. He
should be maintained in a thermoneutral
environment at which the body
temperature is maintained with the least
metabolic and oxygen requirement. This
temperature range is between:
32oC – 35oC (p.247, del Mundo, 4th
ed.)
47. A child is fond of cuddling her pet dog and
cat. She developed fever, lymph node
enlargement at the axillary area, that is
solitary and tender, with no cellulites.
Most likely she has:
Cat-Scratch Disease (p.559, del
Mundo, 4th ed.)
48. The infant with congenital absence of
abdominal musculature presents with a
bulging abdomen, bilateral flank fullness,
and loose wrinkled skin covering the
anterior abdomen has:
Prune-belly syndrome (p.290, del
Mundo, 4th ed.)
49. A child presented with generalized
edema, serum albumin of 15 mg/dL and
cholesterol of 656 ….
Nephrotic Syndrome
50. In a one year old infant with measles, the
WHO recommends this dose of Vitamin
A:
200,000 IU oral, one dose
51. Ipecac is contraindicated in this condition:
Ingestion of caustic substance
52. Toxic myocarditis occurs in 20 % of cases
in:
Diptheria
53. Cerebellar ataxia is most frequently
implicated with this viral infection:
Varicella (p.1072, del Mundo, 4th ed.)
54. Sensorineural hearing loss is most
commonly caused by this viral infection:
Rubella
55. One of this is NOT an isotonic solution:
56. This is given late in the resuscitation:
Defibrillate and give medication
57. Capillary refill is 12 secs. What will you
do?
Give a bolus of isotonic solution at
20 mL/kg.
58. ABCDE of neonatal resuscitation. “E”
stands for:
Thermal environment (p. 268, del
Mundo, 4th ed.)
59. The following are correctly given as
medication in resuscitation EXCEPT:
Epinephrine via ET at 0.1 mg/kg
using 1:10,000 dilution (should be 1:
1000)
60. Which of the following is an effect of the
vasopressor class of cardiac medications:
61. A two day old purely breastfed presented
with bleeding from the nose, umbilicus
and gastrointestinal tract. This infant’s
condition is most likely:
Deficient in Factors VII, IX, X.
(p.286, del Mundo, 4th ed.)
62. A child presented with a ping-pong
deformity. The expected laboratory
findings are: (A case of Ricketts)
63.
64.
65.
66.
Serum calcium level may be normal
or low, the serum phosphorus level is
low, and the serum alkaline
phosphatase is elevated (p.186,
Nelson)
A newborn presented with abdominal
distention and on x-ray of the abdomen
showed a “double-bubble” sign. This
condition is associated with:
Bilious vomiting
A 2 day old presented with jaundice. The
total bilirubin is 12 mg/dL and the indirect
is 7 mg/dL. You will:
Investigate for the cause of the
jaundice.
This is a disorder of movement and
posture caused by a static defect or
lesion of the immature brain:
Cerebral palsy (p. 1093, del Mundo,
4th ed.)
A female pseudohermaphrodite is:
A Virilized female (p.1760, Nelson)
67. A child was bitten by her six year old
sibling. You will give an antibiotic that will
cover for:
Both aerobes and anaerobes (pp.
791-792, Nelson)
68. A son of a farmer presented with fever,
myalgia, headache, abdominal pain,
vomiting and conjunctival suffusion. Most
likely condition is:
Leptospirosis (p.486, Del Mundo, 4th
ed.)
69. A child with leg mass, the most important
to elicit is:
70. Retinoblastoma (RB1)
71. Associated with Varicella with good
prognosis:
72. Renal scarring is best evaluated with:
Renal Scan
73. A child presented to you at the OPD with
a past history of UTI treated with an
antibiotic. His urinalysis now is (+) for
pyuria. You will do:
Congenital lobar emphysema
(p.1274, Nelson)
79. The majority of neonatal infection is
caused by:
GBS
80. Community acquired pneumonia is most
often caused by:
S. pneumoniae
81. True of mycoplasma pneumonia
epidemiology, EXCEPT:
Occurs worldwide
Overt illness is unusual before 3-4
years of age
Endemic in larger communities
Highly communicable (it’s not
highly communicable)
(p. 914, Nelson)
82. This substance produced IUGR, limb
anomalies, autism in the infant:
Alcohol (p. 468, Nelson)
83. True of acute hematogenous
osteomyelitis:
Predilection for the metaphyseal
region (p. 1139, del Mundo, 4th ed.)
84. The most common cause of pleural
effusion is:
Bacterial Pneumonia (p 1329,
Nelson)
85. Inspiratory and expiratory chest film is
most valuable in:
suspected foreign body inhalation
(p 1254, Nelson)
86. Esophageal varices in a patient with liver
cirrhosis, most important modality:
Endoscopy
87. Diagnosis of gastroesophageal reflux
disease:
24 hr. pH monitoring
88. A child who presented at about the 5th day
of illness with a reticular or lacy pattern of rash
on the extremities, most likely has:
Parvovirus infection (p. 534, del
Mundo, 4th ed.)
89. A child presented at the ER with fever,
respiratory distress and croupy cough, most
likely has:
Acute laryngotracheobronchitis (p.
665, del Mundo, 4th ed.)
90. A child with previously treated PPTB and
with healed lesion, developed measles:
Urine culture and sensitivity
91. A child with relapse of PTB:
74. Dribbling and … On ultrasound the
bladder wall is thickened, you will do:
VCUG
75. The best … for infective endocarditis is:
Blood culture
76. One of these is NOT a major criteria in the
Jones criteria for RF:
Arthralgia
77. Apgar score of a newborn at one minute
that has: HR = 95/min, limp, blue, grimace on
catheter insertion, hypoventilating.
APGAR = 3
78. The most common congenital lung lesion
is:
Cystic adenomatoid malformation
(p.651t, del Mundo, 4th ed.)
92. Corticosteroid is beneficial in which forms
of PTB:
All of the above (TB Meningitis,
Pleuritis and Pericarditis)
93. Which should NOT be done in Kerosene
ingestion?
do lavage
94. Congenital TB is rare because:
most common result of female
genital tract TB is infertility (p. 887,
Nelson)
95. A child with asthma with PEFR between
60-80% has:
Moderate persistent asthma (p.27t,
Asthma Consensus, 2002)
96. A child was brought by his mother
because he was exposed to his father with
HAV infection three weeks ago, you will give:
Immunoglobulin at 0.02 mL/Kg and
active immunization
Advise on hygiene
Note: The use of IG more than 2
weeks after exposure is not
recommended
( p.771, Nelson)
97. In a child with tetanus and is allergic to
penicillin, the best option is to give:
Metronidazole
98. Automatism is commonly associated with
this kind of seizure:
Complex partial seizure (p.1036, del
Mundo, 4th ed.)
(p. 1815,
Nelson)
99. A newborn with jaundice has
hyperbilirubinemia and positive Coomb’s test.
He has:
ABO incompatibility
100. Anemia in the newborn is most commonly
due to:
Hemolysis (p.285, del Mundo, 4th
ed.)
101. RDS type II is due to:
delayed absorption of the
respiratory fluid (p.283, del Mundo, 4th
ed.)
102. Endotracheal intubation with application
of negative pressure is beneficial in:
Depressed infant with thick
meconium staining
103. NOT a modality in RDS I:
Nitric oxide
104. Apnea in the newborn is treated with:
Methylxanthines
105. A premature with systemic candidiasis
with renal impairment. The drug of choice is:
Liposomal amphotericin B (p. 933,
Nelson)
Ketoconazole
Fluconazole
106. A 2 year old presented with pallor. Hgb is
7 g/dL. He is asymptomatic. This child is best
treated with:
Oral Iron
Transfusion then oral iron
107. Antidote for Paracetamol overdose:
N-acetylcysteine
108. The ratio of serum Na to serum K is:
109. A child with Cushing’s syndrome has:
Obesity (p.1738, Nelson)
Hypernatremia
Hypertension
110. The earliest sign of puberty in males is:
Testicular enlargement
111. The earliest signs of puberty in Filipino
males and females is seen between:
_________years old
112. A child with recurrent UTI, leg-length
discrepancy, talipes cavus most likely has:
Spina bifida (p.1066, del Mundo, 4th
ed.)
Tethered Cord (p.1864, Nelson)
113. The earliest immunoglogulin to rise after
a recent immunization is:
IgM (p.306, del Mundo, 4th ed.)
114. A 10 year old female with fever and
necrotizing fasciitis, most likely has:
Group A Streptococcus (p. 446t, del
Mundo, 4th ed)
115. Type IV delayed type hypersensitivity is
exemplified by:
Graft vs. host reaction (p. 647,
Nelson)
116. Major criteria for the diagnosis of atopic
dermatitis include: pruritus, typical morphology
and distribution and personal or family history
of atopy. Which is also part of the major
criteria for diagnosing atopic dermatitis?
Tendency to recurrences (p. 416,
del Mundo, 4th ed.)
117. The principal noncytotoxic mechanism for
urticaria and angioedema is:
Interaction of antigen with mast cell
or basophil-bound IgE antibodies
(p.684, Nelson)
118. Central necrosis becoming
vesiculobullous, involves 2 or more mucous
membranes, either eyes, mouth, genitals or
perianal area.
Stevens Johnson (p.1990, Nelson)
119. Characteristic lesion of scabies infection:
Burrow lesion (p.1225, del Mundo,
4th ed.)
120. Drug of choice for rheumatoid arthritis:
NSAID
121. A child has diarrhea with a 4 x 4 cm
palpable mass at left upper quadrant:
Neuroblastoma
Wilm’s tumor
HUS?
122. A child with Acute lymphocytic leukemia
followed up. He has testicular engorgement.
What will you do?
Biopsy (p.1546, Nelson)
123. Reactivity with PPD wanes at about this
age in infants vaccinated with BCG
4-5 years
124. Patients being treated with antibiotics for
Group A Strep pharyngitis which does not
resolve in 24-48 hrs should be suspected of
having:
Infectious mononucleosis (Triad of
pharyngitis, fatigability, generalized
lymphadenopathy p.979, Nelson)
125. A child with cough of 4 weeks in duration
that is paroxysmal and with development of
subconjunctival hemorrhages and petechiae
on the face, most likely has:
Pertussis (p.466, del Mundo, 4th ed.)
126. Earliest response to oral iron therapy is
seen by:
30 days
127. A child with snoring….
Adenoid hypertrophy
128. Symptom is severe, causes child to loose
consciousness:
Cough syncope? or Breatholding?
129. A component of Tetralogy of Fallot,
EXCEPT:
Dextroposition of the major arteries
130. Statutory rape is:
Rape of a minor?
131. Premarital sex is most frequent among:
132. A child with basal meningitis, it is prudent
to start with this regimen:
Antituberculous treatment (p.891,
Nelson)
133. The following are signs of basal skull
fracture, EXCEPT:
Mandibular fracture (p. 1050, del
Mundo, 4th ed.)
134. A 15 month old presented with recurrent
bouts of abdominal pain, diarrhea, later
developed bloody mucoid stools. Most likely
he has:
Intussusception
135. Effective in the destruction of
gametocytes in Malaria:
Primaquine (p.600, del Mundo, 4th
ed.)
136. A newborn infant delivered presented
with absence of suppination, failure to abduct
the arm from the shoulder and to externally
rotate the arm….
Erb-Duschene paralysis (p.491,
Nelson)
137. Highly sensitive in detecting cases of
Primary TB
138. Treatment of helminthiasis:
every 3-6 months (p.583, del Mundo,
4th ed.)
139. Restrictive lung disease has:
reduced lung volume (p.633, del
Mundo, 4th ed.)
140. Physiologically acceptable fluid to be
used in boluses:
Isotonic solution
141. This is the most common valvular lesion
involved in children with rheumatic heart
disease:
Mitral regurgitation (p. 991, del
Mundo, 4th ed.)
142. Which of the following condition is at risk
for developing subacute sclerosing
panencephalitis?
Measles infection before 18 mos.
143. For infants under 1,000 g, the initial
feeding are either half or full-strangth breast
milk or preterm formula at:
5 ml
10 ml (p. 481, Nelson)
15 ml
20 ml
(to be continued)
by:
Ramon C. Santos, M.D.
Pediatrics
St. Luke’s Medical Center
Last updated: March 31, 2003
Jlc
March 2004 PPS Recalled Exam Subjects:
1.
2.
3.
4.
5.
6.
7.
8.
9.
10.
11.
12.
13.
14.
15.
Nagayana Spots
Effect of Chronic intake of drugs
a. phenobarbital with vomiting
c. valproic acid with
hyperactivity
b. phenytoin with gum hypertrophy
Alice in wonderland Syndrome
Dawson encephalitis
Metabolic derangement in Pyloric Stenosis:
hypochloremic metabolic alkalosis
2 year old exposed to a pesticide sprayed in his
room suffering from nausea, vomiting,
jaundice and hepatomegaly.
a. lead
b. mercury
c. arsenic
Toxicity of this substance can be found in gold
mining areas
a. Mercury
c. lead
b. Copper
A 5 year old girl was screened as having a lead
level of 55 ug/ml. Asymptomatic. Normal PE.
What will you do?
a. relocate the girl from the source of lead
b. remove the girl from the source of lead
and observe
c. relocate the girl and admit to a hospital for
IV EDTA therapy
d. relocate the girl from the source of lead
and treat OPD basis with EDTA
Inherited as X-linked disorder and the second
most common cause of mental retardation in
children:
a. Down syndrome
b. Fragile X
c. Cri du chat syndrome
d. XYY Syndrome
Most common gene defect in Trisomy 21:
a. Translocation
b. Mosaicism
You suspect a case of epiglottitis, what will
you do?
a. do direct laryngospcopy
b. do tracheostomy
c. do CXR with lateral neck
d. give immediately a 3rd gen cephalosporin
Which of the following skin disorder is
inherited as autosomal dominant?
a. Pityriasis rosacea
b. Psoriasis
c. Acne vulgaris
d. Seborrheic dermatitis
Preventive pediatrics plays a major role in _ to
prevent significant morbidity and mortality:
a. child abuse
b. nicotine addiction
c. teen pregnancy
The most common cause of neonatal
gastrointestinal obstruction:
a. aganglionic megacolon
b. meconium ileus
The following are useful in differentiating
Hisrchsprung from functional constipation
except:
a. symptoms start from birth
b. rectal biopsy
c. presence of stool in ampulla
d. palpation of feces in the abdomen
16. The SMR of a girl with sparse, straight pubic
hair and papilla and areola form a second
mound: SMR 2
17. Bone aging correlates with pubertal
development
18. The following is an effect of oligohydramnios
except:
a. pulmonary hypoplasia
b.
19. IUGR with Congenital Rubella Syndrome
20. Respiratory syncitial virus
21. A 2 year old male presents with colicky
abdominal pain, vomiting and blood streaked
stools:
a. Meckel’s diverticulum
b. Intussusception
22. Which if the following has a delay in its effect
in the treatment of asthmatics:
a. steroids
b. nebulized budesonide
c. nebulized salbutamol
d. ipratropium bromide
23. A 5 day old neonate was being given
theophylline then presented with irritability
and seizures. Which of the following
substances mimic the effect of theophylline
having the same metabolite?
a. caffeine
b. terbutaline
24. Structures in the newborn postnally closes
except:
a. Patent foramen ovale
b. Patent ductus arteriosus
c. Ductus venosus
d. Infundibular pulmonic stenosis
25. Immunotherapy is based on the following:
a. Type I
b. Type II
c. Type III
d. Type IV
26. Immune system primarily involved in
tuberculosis:
a. macrophage and T-lymphocytes
b. natural killer cells and B-lymphocytes
c. macrophage and natural killer cells
27. A neonate born to mother who is on her third
month of TB chemotherapy. What should be
given to the infant?
a. INH
b. Rifampicin
c. Ethambutol
28. Which of the following is proven to have a
protective effect against otitis media?
a. breastfeeding
29. A 1 year old asthmatic who just completed his
immunization 6 months ago was bitten by a
stray dog. What should be given to the child?
a. tetanus toxoid, tetanus vaccine, HDCV
and ERIG
b. HDVC and HRIG
c. ERIG and HDVC
30. A hyperdynamic precordium signifies which of
the following:
a. right ventricular hypertrophy
b. left ventricular hypertrophy
c. large left to right shunt
31. A status asthmaticus seen at ER in impending
respiratory failure presents with the following
ABG:
a. ph=7.2, pO2=60, pCO2=45
32.
33.
34.
35.
36.
37.
39.
40.
41.
42.
43.
44.
45.
46.
b. ph=7.4, pO2=80, pCO2=40
c. ph=7.2, pO2=100, pCO2=40
Which of the following is a direct
measurement of hemolysis?
a. hematocrit
b. reticulocytosis
c. retic index
Which of the following is a critical level in the
secondary polycythemia in congenital cyanotic
heart diseases?
a. 50% volume
b. 55%
c. 60%
d. 65%
Which of the following is the objective way to
monitor asthma attacks?
a. spirometry
b. peak flow expiratory rate
c. diary of symptoms
Which of the following substances being given
in acute attacks of asthma has an effect of
transient paradoxical hypoxemia?
a. ipratropium bromide
b. inhaled budesonide
c. IM epinephrine
d. Nebulized salbutamol
A NICU with equipment for surgical, pedia
medical represents what level of hospital care?
a. subspecialty care
b. special care
c. basic care
d. intermediate care
A 2 year old presents with hepatomegaly and a
3x3 cms palpable mass at left lower quadrant:
a. neuroblastoma
b. wilm’s tumor
The roentgenographic pattern in osteosarcoma:
a. sunburst pattern
b. onion skinning
The diagnostic of choice in vesicoureteral
reflux:
a. Renal Ultrasound
b. IVP
c. VCUG
The probable cause for a primary reflux:
a. neurogenic bladder
b. cystitis
c. distal bladder obstruction
d. (anatomic)
EEG is indicated in the following:
a. febrile seizure
b. first non-febrile seizure
c. meningitis
d. encephalitis
Which of the following is noted at 9 months?
a. object permanence
Skipping, drawing a triangle, etc is developed
at what age?
a. 5 years old
An infant learns to say “mama, dada” at least
at what month?
a. 8 months
b. 9 months
c. 10 months
d. 12 months
Which of the following developmental aspect
is markedly developed at 2 years old?
a. social
b. cognitive
c. linguistic
47. Breathholding peaks at what age?
a. 6 months
b. 12 months
c. 24 months
d. 36 months
48. Which of the following is the most commonly
used instrument in ‘child abuse’?
a. hand
49. What is the most common cause of death in
child abuse?
a. intentional head trauma
50. What age is high risk for submersion
drowning?
a. 1-4 years
51. 1-4 years old are most prone to:
a. Drowning
52. A 2 day old infant presents with dyspnea.
Chest x-ray showed cystic mass.
a. CCAM
53. A 10 year old presents with low to moderate
grade fever, sudden respiratory distress. PE
showed lagging of the right chest, bronchial
breath sounds below the right scapular area,
dullness on percussion and tympanitic on
auscultation. There is leukocytosis with
neutriphilic predominance.
a. pleural effusion
b. lung abscess
c. pneumothorax
d. consolidation
54. The following characterizes pleural exudate
except:
a. pH > 7.2
b. serum to pleural protein ratio <0.5
a. LDH < 500
55. The diagnostics to use in the diagnosis of UTI
in children:
a. ultrasound of kidney and bladder
b. renal scan
56. It is mandatory to measure BP at what age?
a. 2 years
b. 3 years
c. 5 years
d. 7 years
57. Which of the following characterizes the
physiology in children?
a. RR and heart rate decreases markedly in
the first 2 years of life
b. BP increases starting at 6 years old
c. All of the above
58. Cholesterol screening in adolescents especially
those with a strong family history
59. Most common cause of osteomyelitis in
children:
a. Staphylococcus aureus
b. Staphylococcus epidermidis
c. Streptococcus pyogenes
60. A case of basilar enhancement, hydrocephalus.
No symptoms in the family. What will you
give?
a. anti-Kochs
61. A 10 year old was recently adopted at DSWD.
Record retrieved at health center showed that
he received a DPT and OPV. What will you
recommend?
a. DPT , OPV, MMR and Hepa B
b. DT, OPV, MMR and Hepa B
c. DT, OPV, MMR and Varicella
Pghpeds2003c/olaiza
PPS Specialty Boards Recall Questions – March
2005
1. A 1 year old patient born to an HbsAg (+)
mother. Patient was given HBIg and HBV at
birth; and subsequent HBV doses at 1 and 6
months old. Mother shows you patient’s
hepatitis profile:HbsAg (-), anti-HbsAg (-), antiHBc (-). What do you do?
a. repeat HBV immunization
b. refer to hepatologist
c. allay fears for mother and tell her that her
child will develop antibodies to hepB in 6
months time
d. do nothing
2.
Pineal gland calcifies at:
a. 6 years old
b. 8 years old
c. 10 years old
d. 14 years old
3.
20 primary teeth complete at:
a. 2 years old
b. 3 years old
c. 4 years old
d. 5 years old
4.
5.
6.
Object constancy is achieved at:
a. 7 months
b. 8 months
c. 9 months
d. 10 months
Triad of congenital rubella:
a. cardiac, cataract, deafness
b. blueberry muffin lesions, cardiac, deafness
c. cataract, mental retardation, deafness
Most common presentation of a neonate with
Hirschprung disease:
a. abdominal distention with complete or
partial obstruction
b. delayed passage of meconium
c. sepsis
7.
Cranial tumor with poorest prognosis
a. brainstem glioma
b. craniopharyngioma
c. pinealoma
d. astrocytoma
8.
Important in asthma management according to
the Philippine Concensus for the Management
of Childhood Asthma:
a. regular monitoring of lung function
b. gene therapy
c. exercise challenge
d. cow’s milk allergy
9.
Couple with heredofamilial disease. What do
you do:
a. gene therapy
b. genetic counseling
10. Diagnostic test for pulmonary embolism:
a. ABG
b. CXR
c. ventilation lung scanning
d. pulmonary angiography
11. Diagnostic exam to determine cardiac
involvement in a patient with Kawasaki
disease:
a. CXR
b. 2-D echo
12. Patient with Wilms tumor. Associated finding:
a.
b.
hemihypertrophy
---
13. Complication of TPN:
a. electrolyte imbalance, hypo/hyperglycemia
b. CONS sepsis
c. vitamin deficiency
d. all
14. Vitamin A deficiency, EXCEPT:
a. hyperkeratosis
b. bleeding in subperiosteal areas
c. associated with decreased fat intake
15. Protein requirement in 1-3 year old:
a. 20 g
b. 24 g
c. 30 g
16. Munchaussen syndrome by proxy:
definition ---17. Causes ??? myocarditis:
a. Coxsackie A
b. Coxsackie B
c. Enteroviruses
18. Patient born to an HIV (+) mother. Youngest
age when you can declare patient free of HIV
infection:
a. 6 months
b. 12 months
c. 18 months
d. 24 months
19. Most common injury among toddlers:
a. burn, fall, drowning
20. Most common injury in the home and school:
a. motor vehicular accidents
b. fall
c. burn
21. Drowning patient is asymptomatic after
resuscitation. When will patient become
symptomatic:
a. 4 hours
b. 10-12 hours
c. 12-24 hours
22. Patient with hypopigmented lesion has
seizures:
a. Tuberous sclerosis
b. Sturge Weber
c. Incontinentia Pigmenti
d. Hemangiofibroma
23. Lead poisoning is treated if:
a. serum lead levels > 45 mg/dl
24. 3-year old patient took 30 ml of 250mg/5ml
Paracetamol. Which is correct:
a. patient took toxic dose
b. antidote is N-acetylcysteine
c. give syrup of ipecac to induce vomiting
25. Lab exam to determine long-term control of
glucose:
a. FBS
b. HbA1c
c. urine glucose
26. Definitive diagnosis for patient with TB
meningitis:
a. CT scan showing basal enhancement and
infarction
b.
positive TB culture
27. Statistic on VCUG and VUR:
28. Foul-smelling nasal discharge:
a. chronic sinusitis
b. diphtheria
29. Patient took metoclopramide and later
manifested with choreoathetosis. What is the
drug of choice:
a. diphenhydramine
30. Inactivated toxin:
a. toxoid
31. Erythroblastosis fetalis is what type of
hypersensitivity:
a. type I
b. type II
c. type III
d. type IV
32. Most useful tool in allergy:
a. skin test
b. serum IgE
c. RAST
33. Associated with Turner syndrome:
a. Coarctation of the aorta
b. Endocardial cushion defect
c. PDA
d. VSD
34. Associated with Down syndrome:
a. VSD
b. Endocardial cushion defect
c. PDA
d. Coarctation of the aorta
35. Presence of blowing holosystolic murmur on
the left parasternal border in a patient who is
cyanotic when crying:
a. VSD
b. PS
c. PDA
36. Patient with large VSD and pulmonary
hypertension. When is the best time to
operate:
a. 6-12 months
b. 18-24 months
37. Pathognomonic sign on x-ray in necrotizing
enterocolitis:
a. pneumatosis intestinalis
b. pneumoperitoneum
38. Patient with webbing of the neck, low posterior
hairline, and lower extremity pulses weaker
than the carotids:
a. coarctation of the aorta
39. Congenital obstruction of the GI tract is
characteristic of what type of amniotic fluid:
a. anhydramnios
b. oligohydramnios
c. polyhydramnios
40. 6-hour old infant with hematemesis and
hematochezia. This is due to maternal intake
of:
a. phenobarbital
b. valproic acid
c. lithium
d. prednisone
41. 12-hour old term infant previously doing well
experiences respiratory distress. Patient has
soft systolic murmur and has received 3 feeds
for the past 12 hours. (+) maternal history of
PROM of 18 hours. Which is the least likely
cause:
a. TTN
b. aspiration pneumonia
c. sepsis
d. congenital heart disease
42. 5mm induration is considered negative in:
a. asymptomatic patient in endemic area
b. immunocompromised patient
43. G-6-PD (+) baby. Tell mother:
a. MR is common
b. MR occurs in those with hyperbilirubinemia
and kernicterus
c. MR occurs even without jaundice
44. Bird flue strain:
a. H1N2
b. H2N5
c. H5N1
d. H7N3
45. SARS is due to :
a. coronavirus
b. rhinovirus
c. echovirus
46. Congenital rubella triad:
a. blueberry muffin lesions, cataract,
deafness
b. microcephaly, cataract, deafness
c. congenital heart disease, cataract,
microcephaly
47. Chorioretinitis, hydrocephalus, and cataract
comprise the triad of:
a. toxoplasmosis
b. congenital rubella
c. CMV
48. Patient with vomiting, nausea, and histologic
finding of decreased ganglionic cells
a. chalasia
b. achalasia
49. Normal anion gap is found in:
a. RTA
b. DM
c. uremia
d. nephrotic syndrome
50. Persistent hepB is best characterized as:
a. persistent HbSAg
b. persistently elevated bilirubin and liver
enzymes after acute hepatitis
51. Generalized jaundice in a 1-day old.
Determine:
a. serum bilirubin levels
b. mother’s BT
c. baby’s BT
d. mother and baby’s BT
52. Persistent toe-walking is a sign of which
developmental delay:
a. neurologic
b. motor
c. language
53. Adolescent with history of respiratory infection
followed after 1 week by hematuria and
hemoptysis. (-) dsDNA. What is the
diagnosis:
a. SLE
b. Goodpasture’s syndrome
54. Patient with intermittent polyuria and
polydipsia. Lab test needed:
a. urine culture
b. urinalysis and FBS
c. urinalysis and GTT
55. Patient with increased skull AP diameter. This
signifies premature closure of which suture:
a. coronal
b. sagittal
c. lambdoid
56. Viral syndrome with cicatricial --a. varicella
b. rubella
c. CMV
57. Adolescent with hyperpigmented lesions on the
mouth experiences crampy abdominal pain.
a. Peutz-Jeghers syndrome
b. Adenopolyposis coli
c. Gardner syndrome
58. Total caloric requirement should make
allowances for all of the following EXCEPT:
a. basal metabolic rate
b. emotional distress
c. physiologic growth
59. Indirect bilirubinemia is found in:
a. hypothyroidism
b. breastmilk jaundice
c. sepsis
60. Which of the following is not a risk factor for
RDS:
a. asphyxia
b. maternal pre-eclampsia
c. prematurity
d. Cesarean delivery
61. Most common cause of prolonged fever:
a. bacterial infection
b. connective tissue disease
c. malignancy
62. 2-week old patient presenting with opisthotonus
and seizures. What do you ask:
a. feeding history
b. manner of delivery
c. maternal history
63. Pineal gland calcifies at what age:
a. 6 years old
b. 8 years old
c. 10 years old
d. 12 years old
64. Produced in large quantities, neutralizes
viruses:
a. IgM
b. IgG
c. IgA
d. cytokines
65. Patient with congenital hydrocephalus. Most
economical diagnostic test:
a. cranial ultrasound
b. cranial CT scan
66. Used in diagnosing SLE, EXCEPT:
a. ANA
b.
c.
d.
CBC
2-D echo
C3
67. Principle of single-dose aminoglycoside
a. higher peak than MIC results in bacterial
killing
b. prolonged exposure results in bacterial
killing
68. Localize sound and familiar names
a. 6 months
69. 14-year old with TB Meningitis stage III. Pullsout NGT and says she doesn’t want it
reinserted and no longer wants to live. What
do you do:
a. reinsert NGT anyway
b. comply with request
c. refer to ethics committee
70. Intubated newborn with Down syndrome and
Hirschsprung. Mother tells that she wants to
withdraw life support. What do you do:
a. comply with request
b. continue management and refer to
surgeon
c. consult other members of the family and
get consensus
d. refer to ethics committee
71. Best way to prevent bicycle accident:
a. do not let the child ride a bicycle
b. use of helmet when riding bicycle
72. Patient with chronic diarrhea. Patient infected
with:
a. Amoeba
b. Enterobius
c. Giardia
d. Trichuris
73. Suggest Wilms Tumor
a. hemihypertrophy
74. Patient with abdominal pain and currant jelly
stools
a. intussusception
75. Indicative of Hirschsprung Disease:
a. absence of meconium
76. Patient with DHF, platelet of 80,000, and blood
nasal discharge. Maintenance fluid to use:
a. D5LR
NOVEMBER 2005 RECALL
1.
Follows moving objects 180º, smiles on social
contact, listens to voice and coos
a. 2 months old
b. 4 months old
c. 6 months old
d. 8 months old
a.
b.
c.
d.
2
3
4
5
2.
Walks up and down the stairs
a. 1 year old
b. 1 ½ years old
c. 2 years old
d. 2 ½ years old
12. 10-year old boy with short stature, normal
perinatal history, (-) illnesses; has always been
smaller than other boys his age; growth is
below and parallel to the 3rd percentile; bone
age is consistent with height age
a. constitutional growth delay
b. growth hormone deficiency
c. pathologic short stature
d. nutritional insufficiency
3.
Lifts head and chest with arms extended, tonic
neck posture predominates, reaches toward and
misses objects, waves at toy, listens to music
a. 4 weeks old
b. 8 weeks old
c. 12 weeks old
d. 16 weeks old
13. The mom of a preterm infant would like to be
transferred to a charity hospital since she does
not have any money. What do you do?
a. give oxygen inhalation and
thermoregulate
b. start IV fluids
c. do nothing
4.
Imitates a triangle, names 4 colors, counts 10
pennies, dresses and undresses, asks questions
about meaning of words
a. 2 years old
b. 3 years old
c. 4 years old
d. 5 years old
14. Giving pain meds to a terminally-ill patient
a. palliative care
5.
6.
7.
8.
9.
Sits up alone and indefinitely without support,
back straight, plays peek-a-boo, waves bye-bye
a. 8 months old
b. 10 months old
c. 12 months old
d. 15 months old
Feeds self, seeks help when in trouble, kisses
parent with a pucker, identifies 1 or more parts
of the body
a. 12 months old
b. 15 months old
c. 18 months old
d. 24 months old
Appearance of mandibular central incisors
a. 5-7 months old
b. 8-10 months old
c. 10-16 months old
d. 16-20 months old
Sleep requirement in hours per day of a 3month old
a. 17
b. 15
c. 13
d. 11
Conformity to peers is important
a. preschool
b. middle childhood
c. early adolescence
d. late adolescence
15. Iron is 2-3 times absorbed more readily in
which of the following:
a. cow’s milk
b. breastmilk
c. skimmed milk
d. hypoallergenic milk
16. What is the recommended age to stop
breastfeeding?
a. 6 months
b. 10 months
c. 12 months (?)
d. 18 months
17. Absolute contraindication to breastfeeding
18. Pica and anemia
a. folate deficiency
b. iron deficiency
c. vitamin B12 deficiency
19. A patient with cough, colds, and fever with
diplopia, papilledema, and cranial palsies. You
suspect:
a. Vitamin A intake >1500 IU per day
b. Vitamin C deficiency
20. Another question on vitamins
21. The following are associated with Vitamin A
deficiency except:
a. silver-gray plaques on the
conjunctiva
b. dry and scaly skin with follicular
hyperkeratosis
c. fat malabsorption
d. epiphyseal enlargement of wrists
and ankles
10. Consolidation of sexual identity
a. middle childhood
b. early adolescence
c. middle adolescence
d. late adolescence
22. Sodium content of isotonic solution in meqs/L
a. 154
b. 130
c. 77
d. 35
11. Tanner stage: previously scanty hair increased
in amount and begins to curl, penis longer,
testes larger
23. Question on glucose infusion rate. Know the
formula.
24. True about total body water
a. term infants have a higher total body
water than preterms
b. term infants have 90% total body
water at birth
c. during the 1st year of life, total
body water decreases to 60% of
body weight and remains at this
level until puberty
d. males have more total body water
than females
25. True about osmolality except:
a. ECF and ICF are in osmotic
equilibrium because the cell
membrane is freely permeable to
water
b. A shift of water into the ECF
occurs as the ICF osmolality
increases
c. An increase in ECF osmolality
causes a shift of water out of the ICF
d. Plasma osmolality is 285-295
mOsm/kg
26. Decreased serum Na, decreased to normal
urine output with increased urine Na and
osmolality
a. SIADH
b. Cerebral salt wasting
c. dehydration
27. The following are causes of metabolic
acidosis except:
a. RTA I
b. RTA II
c. diuretics
28. What is the best advice to give to parents on
how to avoid swimming pool accidents?
a. keep the pool covered
b. build a shallow pool
c. put a fence around the pool
d. always have a watcher present
29. Toddlers are at least risk for:
a. falls
b. poisoning
c. firearm-related injuries
d. drowning
30. Most common accident among 2-year olds
a. fall
b. burns
c. suffocation
d. poisoning
31. Most common cause of serious injuries in
motor vehicle accidents (sorry can’t remember
the other choides)
a. ejection
a.
b.
c.
Furosemide
Mannitol
Acetazolamide
34. Normal capillary refill time in seconds
a. 2
b. 4
c. 6
d. 8
35. True about x-linked recessive diseases
a. man transfers the gene to 50% of his
daughters
b. heterozygous female carriers are
unaffected
c. may be transmitted from father to
son
36. The following are components of the NBS
except:
a. CAH
b. CH
c. PKU
d. MSUD
37. The following are conditions associated with
polyhydramnios except:
a. Ileal atreasia
b. Omphalocoele
c. Renal agenesis
d. Duodenal atresia
38. Infants weighing <1000g can lose as much as
a. 7-8 ml/kg/hr
b. 6-7 ml/kg/hr
c. 4-5 ml/kg/hr
d. 2-3 ml/kg/hr
39. Fluid requirement of a term infant on D2-D3 of
life
a. 20-30 ml/kg
b. 50-60 ml/kg
c. 70-80 ml/kg
d. 100-120 ml/kg
40. The most common site of hemorrhage in
preterms
a. subdural
b. subarachnoid
c. intracerebral
d. intraventricular
41. The site of hemorrhage in a term infant
delivered after a difficult labor
a. subdural
b. subarachnoid
c. intracerebral
d. intraventricular
42. LGA infant born via SVD with note of
torticollis - due to traumatic rupture of SCM
32. Scale used to assess patients with altered level
of consciousness
a. Pediatric Trauma Score
b. Glasgow Coma Score
33. Which of the following relieves increased
intracranial pressure by decreasing blood flow
to the choroidal arteries?
43. A preterm baby weighing 2000 grams
presented with cyanosis and retractions on the
2nd hour of life
a. cyanotic heart disease
b. HMD
44. Which of the following can be used to
distinguish between GBS and HMD?
a. asymptomatic bacteremia
b. well at birth
c. apnea with early shock
45. A decreased risk of HMD occurs in
a. Maternal use of antihypertensive
medications
b. Administration of antenatal
steroids
46. Pneumatosis intestinalis may be found in
which of the following:
a. NEC
b. Ileus
47. Management of a 2,500 gram infant who
develops jaundice within the first 24º of life;
the mom is type O while the baby is type A;
(+) Coombs, TB 20, IB 18
a. start IVF and phototherapy
b. exchange transfusion
c. repeat bilirubin determination after
36º
d. do nothing
48. Polycythemia may be seen in the following
conditions except:
a. twin-twin transfusion
b. maternal-fetal transfusion
c. SGA
d. congenital spherocytosis
49. A baby was born with short palpebral
fissyures, epicanthal folds, maxillary
hypoplasia, micrognathia, decreased weight,
length, and head circumference. These findings
are consistent with:
a. fetal alcohol syndrome
b. Trisomy 21
c. Trisomy 18
50. The following causes withdrawal symptoms of
tachycardia and hyperactivity except:
a. alcohol
b. barbiturates
c. opioids
d. salicylates
51. Which of the following produce withdrawal
symptoms of insomnia, “goose flesh”,
tachycardia, and diarrhea?
a. opioids
b. salicylates
52. Gastric lavage may NOT be done in which of
the following?
a. caustic ingestion
b. chalk ingestion
54. Which of the following documents allergyspecific IgE in serum?
a. RAST
b. Skin test
c. Skin prick/puncture technique
d. Methacholine challenge test
55. Anterior uveitis may be seen in:
a. JRA
b. SLE
c. Juvenile dermatomayositis
d. Scleroderma
56. Abdominal pain, purpura, joint pains
a. Kawasaki
b. HSP
57. Pulseless disease with erythema nodosum and
malar rash
a. Takayasu arteritis
b. SLE
c. PAN
d. HSP
58. The tips of the fingers initially appear violet
becoming purpuric; with intermittent
abdominal pain, hematuria, hypertension, and
proteinuria
a. periarteritis nodosa (don’t know if
this is a typo error and should be
polyarteritis nodosa)
b. HSP
c. IE
59. 15-year old female from the US with fever; the
patient is a tampon user (also with other
symptoms that I can’t remember)
a. TSS
b. Streptococcal toxic shock syndrome
60. Strep B is a common cause of bacteremia,
meningitis, and pneumonia among children < 2
years old because of:
a. inability to form defenses
against polysaccharide capsulecontaining organisms
61. Which of the following is the drug of choice
for single dose treatment of Gonococcal
infection?
a. Ceftriaxone
b. Penicillin G
c. Cefotaxime
d. Erythromycin
62. Pertussis
a.
b.
c.
d.
paroxysmal cough with sore throat
paroxysmal cough with wheezes
paroxysmal cough without fever
paroxysmal cough with hoarseness
63. Your suspicion of Salmonella enteritis is
further supported by:
a. exposure to a classmate with
gastroenteritis
b. intake of a hotdog
53. Case of a girl with recurrent, prolonged cough
with pruritic, crusting lesions on the lower
extremities
a. work-up for immunodeficiency
64. A patient with fever, abdominal pain, and
bloody diarrhea
a.
b.
S. dysenteriae serotype 1
Shiga toxin-producing E. coli
65. Despite disinfecting hospital equipment,
nosocomial infection by Pseudomonas still
occurs because of:
a. ingestion of contaminated
secretions from the gastrointestinal
tract
b. frequent use of povidone iodine
c. the use of infected linen
66. Management of a baby born to a mom with
suspected MDRTB
67. Management of a baby born to a mom with
untreated tuberculosis
68. Which of the following is the most common
mode of Syphilis transmission among
children?
a. vertical
b. direct contact
c. sexual contact
d. blood transfusion
69. Management of a neonate with (+) VDRL and
RPR whose mother is positive for syphilis and
was treated with penicillin for 6 months
starting at 6 months AOG
70. Cold agglutinins
a. Chlamydia
b. Mycoplasma
71. Which of the following is the most common
infection caused by Candida in
immunocompetent children?
a. Diaper dermatitis
b. Oral thrush
c. Periungual infections
d. Vulvovaginitis
72. Hecht pneumonia
a. Rubella
b. Rubeola
c. Chlamydia
d. Varicella
73. Papular-pruritic gloves and socks
a. Echovirus
b. Coksachievirus A
c. Parvovirus B19
d. Coksachievirus B
74. What is the route through which Naegleria
reaches the brain?
a. oral
b. nasal
c. hematogenous
d. lymphatics
75. Which of the following is true about Giardia?
a. Least common protozoal infection in
the US
b. Diagnosed by finding cysts,
trophozoites, antigens in stool
c. Boiling is ineffective for inactivating
cysts
d. Symptomatic infection occurs more
frequently in adults than in children
76. Algid malaria
a.
b.
c.
d.
Plasmodium ovale
Plasmodium vivax
Plasmodium malariae
Plasmodium falciparum
77. Tramway sign
a. Ascariasis
b. Trichuriasis
c. Enterobiasis
d. Strongyloidiasis
78. Katayama fever
a. Strongyloidiasis
b. Filariasis
c. Schistosomiasis
d. Trichinosis
79. Vaccine that can be given on the first visit Hepatitis B
80. Management of a neonate born to a mother
who is HBsAg positive
a. give vaccine and immunoglobulin
within 12 hours
81. HbeAg is obtained to determine the presence of
a. acute infectious state
b. perinatal infection
c. resolved infection
d. acute hepatitis
82. What is the optimal time for surgery of biliary
atresia
a. 8 weeks
b. 10 weeks
c. 12 weeks
d. 14 weeks
83. Which of the following presents with painless
rectal bleeding?
a. Meckel diverticulum
b. Intussuseption
c. Anal fissure
d. Volvulus
84. Direct bilirubinemia with bile duct paucity,
broad forehead, mandibular hypoplasia, and
peripheral pulmonic stenosis
a. Byler
b. Alagille
c. Dubin-Johnson
85. A condition with agangliosis more commonly
seen in adolescents and young adults
presenting as dysphagia for solids and liquids
with failure to thrive
a. Achalasia
b. Chalasia
c. Barrett esophagus
d. Hiatal hernia
86. Persistent respiratory distress after intubation
in a baby with a scaphoid abdomen
87. A patient with suspected diaphragmatic hernia
still has a pO2 <50 on 1.0 FiO2 after
resuscitation. This is probably secondary to:
a. CHD
b. Pulmonary hypoplasia
88. A patient with fever, dysphagia, bulging of the
peritonsillar wall with displacement of the
uvula
a. retropharyngeal abscess
b. peritonsillar abscess
c. lateral pharyngeal abscess
89. Management of a child who comes to the ER
in respiratory distress and with findings
suggestive of epiglottitis:
a. intubate ASAP
b. start corticosteriods
c. request for radiologic studies
d. nebulize with racemic epinephrine
90. Diagnostic of choice for bronchiectasis
a. bronchogram
b. bronchoscopy
c. fluoroscopy
d. high-resolution CT scan with thin
cuts
a.
b.
c.
acute blood loss of >25% of the
circulating volume
<100 g/dl in the perioperative period
prior to major surgery
99. Philadelphia chromosome
a. ALL
b. AML
c. CML
d. Juvenile CML
100. Management of an infant with a hemangioma
on the thigh
a. excise
b. observe
c. start steroids
d. give interferon-alpha
91. A 13-year old with asthma came to the ER
speaking in phrases, agitated, with HR 100, RR
30, with retractions and wheezes all over, and
PEF 79%
a. intermittent
b. mild persistent
c. moderate persistent
d. severe persistent
101. Renal biopsy reveals mesangial IgA deposits in
the glomerulus similar to those in IgA
Nephropathy
a. Alport
b. SLE
c. HSP
d. HUS
92. Drug of choice in acute exacerbation
a. IV steroids
b. IV theophylline
c. Inhaled steroids
d. inhaled beta2 agonist
102. The edema in Nephrotic Syndrome is
secondary to
a. renal failure
b. salt retention
c. hypoalbuminemia
93. Management of a patient who is a known
asthmatic whose chest x-ray revealed a 5%
pneumothorax
a. 100% oxygen
b. beta2 agonist
c. thoracentesis
d. chest tube thoracotomy
103. More sensitive than IVP in detecting renal
scars
a. DMSA scan
94. 15-year old athlete with sudden chest pain and
abdominal pain; (+) infraclavicular crepitations
a. aortic aneurysm
b. pneumomediastinum
95. Which of the following is the least common
cause of infective endocarditis?
a. Strep viridans
b. Staph aureus
c. Strep fecalis
d. Haemophilus influenzae
96. Which of the following is the most common
cause of myocarditis?
a. diphtheria
b. coxsackievirus B
c. echovirus
d. leptospirosis
97. Worsening of physiologic anemia has been
proved to be associated with
a. Vitamin C deficiency
b. Folate deficiency
c. Vitamin E deficiency
d. Vitamin A deficiency
98. Blood transfusion in older children and
adolescents is warranted in which of the
following situations:
104. Which of the following supports a diagnosis of
hypoglycemia secondary to hypopituitarism?
a. microphallus
b. glucagon test
105. Patients with Turner syndrome should be
referred to which subspecialty for a more
complete follow-up?
a. Cardiology
b. Endocrinology
c. Genetics
106. CSF picture in viral meningitis
a.
b.
b.
c.
lymphocytic pleocytosis, normal protein,
normal glucose
lymphocytic pleocytosis, elevated protein,
normal/decreased glucose
neutrophilic pleocytosis, normal protein,
normal glucose
neutrophilic pleocytosis, elevated protein,
normal/decreased glucose
107. Most common cause of sensorineural deafness
a. bacterial meningitis
b. viral meningitis
108. Triad of osteogenesis imperfecta
a. low set ears, blue sclerae, deafness
b. fragile bones, blue sclerae,
deafness
c.
d.
e.
PPS EXAM NOVEMBER 2003
B
1.
Saxitoxin is associated with
b. paralytic shellfish poisoning
D 2. Permanent hearing loss with reversible
renal damage
c. Furosemide
d. Aminoglycoside
mucosal secretions
bronchospasm
B 13. 2 year old girl with proteineria 2 gm/24
hrs, edema and hyperlipidemia
a. biopsy
b. steroids without biopsy
c. furosemide
d. steroids + cyclophosphamide
B 3. Antihypertensive that causes
hyperkalemia
a. Propranolol
b. Captopril
c. Furosemide
B
14. Friend asks for medical certificate
a. will give certificate for the sake of
friendship
b. explain that you cannot give false
certificate
D
D
15. Pathogenesis of bronchiolitis, except
a. bronchiolar obstruction is due to
edema
b. accumulation of mucus and cellular
debris
c. resistance in small air passages is
increased during inspiratory and
expiratory phases
d. repair of granulation
4.
Procedure to diagnose renal scarring
a. VCUG
b. IVP
c. UTZ
d. DMSA
B 5. Drug which causes hypercalciuria and
nephrocalcinosis
a. Chlorothiazide
b. Furosemide
c. Propranolol
C
6.
Rheumatic fever major criteria except
a. carditis
b. chorea
c. fever
d. polyarthritis
D
7.
Minor criteria for rheumatic fever except
a.  ESR
b.  PR interval
c.  ASO
d. subcutaneous nodules
C
8.
Non-invasive diagnostic procedure
a. digital subtraction angiography
b. cardiac catheterization
c. 2D echo with color flow doppler
d. selective angiography
A
9.
Management of croup
a. cold steam
b. bronchodilator
c. sedative
d. morphine
10. Fear of death
a. 4 – 7 years old
b. 7 – 12 years old
c. 12-15 years old
d. 15-18 years old
C 11. Transfers objects from hand-to-hand, sits
without support, close-open
a. 4 – 6 months
b. 8 – 9 months
c. 10-11 months
d. 12-15 months
B
12. Pathogenesis of asthma
a. edema
b. inflammation
C 16. Na 155, K 5, Cl 110, HCO3 15; compute
anion gap
a. - 12
b. 20
c. 35
d. 60
B 17. 4 year old with Na 125, to increase to 132
mEqs/L, compute for Na correction
b. 67 mEqs
B 18. Mother with (+) PPD without signs &
symptoms
a. isolate baby
b. BCG
c. Do not isolate, do not treat,
investigate other family members
d. Treat with anti-TB meds
D 19. 2 year old exposed to father diagnosed
with PTB 2 weeks ago
b. Xray, PPD then treat if positive
c. Isolate from father
d. Start INH regardless of PPD & chest
Xray
D
20. Significant 10 mm PPD
a. 3 year old with BCG
b. 6 year old with mother with (+) PPD
c.
d. 8 year old previously with 0 mm
D
21. Significant PTB exposure except
a. father in jail but asymptomatic
b. mother with (+) PPD
c. uncle with hemoptysis and fever
d. child with PTB
C
22. Highly infective Hepatitis B
a. IgM anti-HBc
b. HBsAg
c. HBeAg
d. HBcAg
D 23. Chronic subglottic stenosis, most common
cause
a. Tuberculosis
b. GER
c.
c. neonatal intubation
D 24. Case of H. influenza meningitis,
characteristics, except,
a. common in < 2 years old
b. active vaccination encouraged
c. prophylaxis for household contacts
d. steroids x 2 days
A 34. Autoimmune disease have this
complement abnormality
a. C1qrsC4C3
b. C1
c. Membrane attack complex
d. Properdin
A 35. Preterm with sepsis, developed GI
bleeding
a. stress gastritis
b.
b. intussusception
c. H. pylori associated duodenitis
36. Hemolytic disease of the newborn
a. ABO more antigenic than Rh
b. Can develop hydrops fetalis
B 25. Vaccination with whole cell pertussis
prevents
a. mild cough
b. paroxysmal cough of 14 days duration
c. B. pertussis infection
d. B. parapertussis infection
C 26. Puncture wound through rubber shoes is
associated with
a. Group B strep
b. Staph aureus
c. Pseudomonas aeruginosa
27. 5 year old with ALL has fever, ANC <
100 with cultures after 1 days showed
______ should be given _____
a. C. albicans – Ampho B
b. Staph. aureus – Vancomycin
c. Pseudomonas – Cefuroxime
d. Pseudomonas – Cefotaxime
28. H. pylori, diagnostic of choice
a.
b. urea breath test
c.
d. gastroduodenoscopy with biopsy
B 29. Bronchial asthma, greatest risk for status
asthmaticus
a. early use of steroids
b. over-dependence on bronchodilators
c. FEV1 > 80% of expected after agonists
d. Pulsus paradoxus 10 mm Hg
C
C
30. Notching of rib seen in
a. ASD
b. VSD
c. COA
d. TGA
B 38. When to do serum Hgb, Hct in a preterm
infant
a. 4 weeks
b. 8 weeks
c. 9 months
d. anytime
B
39. Nadir of Hct
a.
b. 3 months
c.
d. 7 years
C
40. Majority of Hgb at birth
a. Hgb A
b. Hgb B
c. Hgb F
d. Gower
B 41. Immunoglobulin which crosses the
placenta
a. IgA
b. IgG
c. IgM
d. All of the above
B
31. Egg-shaped heart
a.
b. VSD
c. TGA
d. TOF
A. 32. Resection-anastomosis is procedure of
choice
a. COA
b. PDA
D
37. Abnormal blood picture
a. Hgb 120 mg/d at birth
b. Hgb 8.5 g/d at 5 weeks, infant
preterm
c. Hgb 11 in a 7 year old
d. Hgb 12 in a teen-age girl
33. SLE with (+) Coomb’s test
d. hemolytic anemia
42. True of restrictive lung disease
a. rapid and deep breathing
b. increased load carried almost
exclusively by inspiratory muscles
c. increased elastic recoil of the lung
increases the relaxation volume
d. recoil forces transmitted through the
fibrous network are maximal at low
lung volumes
D 43. Child playing outside was bitten and
developed urticaria, what to do
a. steroids
b. salbutamol nebulization
c.
d. aqueous epinephrine IM
B
44. Most common cause of urticaria
a. malignancy
b.
c.
d.
drugs
infection
parasitism
45. Causes of IgE mediated urticaria (except?)
a. radioactive contrast
b. aspirin
c. penicillin
d. thiamine
46. High-grade proteinuria (albumin +2)
a. high protein intake
b. exercise upon waking
c. supine
d. fever ≥ 38.3C
A 47. Most common cause of serious
intracranial injury in children less than 1 year old
a. fall
b. sexual abuse
D 48. Mother infected with varicella early in
gestation causing serious problems; advise to her
regarding
chances of serious infection
a. 75%
b. 50%
c. 25%
d. <5%
A
49. Most common sign of physical abuse
a. bruises
C
50. Rubella, as compared to CMV, has
a. retinopathy
b. deafness
c. cardiac malformations
B
51. Most common presentation of rubella
b. IUGR
A 52. Neonate with CMV with mild
hepatomegaly, all other tests normal; most likely to
develop within
1 year
a. hearing loss
b. retinopathy
c. brain loss
d. loss of immunoglobulins
C 53. Most severe form of malaria in immunized
persons
a. P. malariae
b. P. vivax
c. P. falciparum
d. P. ovale
D 54. Most lethal infection in
immunocompromised patients
a. M. avium
b. CNS crytococcal encephalopathy
c. Candidiasis (oral)
d. P. carinii pneumonia
A 55. Absolute contraindications in
breastfeeding, except
a. thyrotoxicosis
b. psychiatric problems
c. open TB
d. mother infected with syphilis
56. First dental check-up
a. 6 months
b. 12 months
c. 18 months
d. 2 years old
A
57. Pathogenesis of carious teeth
a. frequent feeding with nursing bottle
b. breastfeeding per demand
c. increased sweetened juice
A
58. Contraindicated in breastfeeding
a. chloramphenicol
b. prednisone
c. furosemide
d. diuretics
C 59. Colostrum compared to mature milk,
except
a. inc. protein
b. dec. fats and carbohydrates
c. less salt and minerals
d. inc. immunoglobulins
D
60. Cerebral palsy, except
a. spastic hemiplegia
b. mental retardation
c.
d. progressive encephalopathy
D 61. Characteristic of benign febrile
convulsions, except
a. < 5 years old
b. with fever
c. family history
d. requiring prophylactic
anticonvulsants
C
62. Absence seizures, except
a. rarely > 30 secs
b. loss of consciousness
c. post-ictal state
d. 3/sec spike
C
63. Most common soft tissue tumor
c. rhabdomyosarcoma
C
64. Access to subarachnoid space
a. subdural tap
b. cranial UTZ
c. lumbar puncture
d. ventricular tap
A 65. Most common presentation of
retinoblastoma
a. leukocoria
C 66. Cannot be differential for
hypomagnesemia
a. hyponatremia
b. hypokalemia
c. hypocalcemia
d. hypophosphatemia
B
67. Simple test for strabismus
a. red eye reflex
b. cover test
c. slit lamp test
A
68. Contraindicated in breastfeeding
a. thiouracil
b.
c.
d. digitalis
69. True about breastmilk
a. dependent on maternal diet
b. fatty acid content depends on
maternal diet
c. mother who is undernourished
without significant difference in fat
content but less in amount
d. varies everyday and with every meal
D 79. Passage of dark, brown hematemesis,
except
a. esophageal varices
b. peptic ulcer
c. esophagitis
d. gastric outlet obstruction
80. Regurgitation, all except
a. effortless, nonforceful movement of
stomach contents into esophagus and
mouth
b. improper feeding technique
c. may be due to mechanical obstruction
in the esophagus
d. associated with failure to thrive
A. 70. 14 year old adolescent took white tablets
presented with lethargy, seizures, metabolic
acidosis and
coma. Patient took
a. isoniazid
b. aspirin
c. acetaminophen
A 71. Phenotypic expression depends on parent
of origin or the gender of the parent from whom it
was
derived refers to:
a. genomic imprinting
b. deletion
c.
d. genetic engineering
A 72. In aneuploidy states, the most common
form of mental retardation occurring in 1:7001:1000
a. Trisomy 21
b. Fragile X
c. Trisomy 18
d. Trisomy 13
A
73. True of NEC, except
a. gastric decompression and lavage
b. occurs in prematures only
c. serial abdominal x-rays
D 74. Embryonic origin, cystic with blood
supply form the systemic circulation
a. bronchogenic cyst
b. CCAM
c. Pulmonary hypoplasia
d. Pulmonary sequestration
75. In patients with increased risk for atopy,
what to advise
a. breastfeeding for 6 months
b. delay solid foods until 9 months
c. use hydrosylated milk formula
A 76. Mechanical, unobstructive causes of
vomiting
a. GER
b. Pyloric stenosis
c. Intussusception
d. Hirschprung’s disease
A
77. Diagnostic of choice for GER
a. 24-hour pH monitoring
b. barium swallow
78. Question on achalasia
81. Toxicity of drugs 2º to passage through
blood-brain barrier and blood-ocular
barrier
a. immaturity of the barrier
b. decreased permeability
c.
d. alkalosis
C
82. Cause of neonatal hypoglycemia in DM
a.
b. pancreatic hypoplasia
c. maternal hyperglycemia leading to
hyperinsulinism
d.
D
83. IUGR 2º to
a. transient neonatal hypoglycemia
b. pancreatic hypoplasia
c. maternal malnutrition
d. all of the above
C 84. Goitrous enlargement in neonates with
hypothyroidism
a. thyroid dysgenesis
b. 1º hypopituitarism
c. thyroid dysmorphogenesis with intake
of maternal thyroid drugs
B
85. 36 weeks, 4.5 kg, APGAR score 3/5
a. PT, SGA, non-asphyxiated
b. PT, LGA, asphyxiated
c. PT, SGA, asphyxiated
d. PT, LGA, non-asphyxiated
C 85. 41 week old newborn with pea-soup
consistency of amniotic fluid
a.
b. ambubagging
c. direct visualization and suctioning
d.
A
86. Hematuria with sensorineural hearing loss
a. Alport’s syndrome
D
87. L-sided Bell’s palsy, except
a. drooping of left lip
b. inability to close left side of the face
c. decreased sensation on the left side of
the face
d. loss of taste of the posterior 1/3 of the
tongue
A 88. Sea-water drowning electolyte
abnormalities
a. hypernatremia and hyperosmolarity
b. hypoosmolarity
c. hyponatremia
D 89. Asperger syndrome, similar to autism,
except
a. social interaction
b. repetetive movements
c.
d. language
C 90. Kwashiorkor and marasmus, same except
marasmus has
a. decreased appetite
b. failure to gain weight
c. muscle wasting
d. normal amino acid levels
D
C
91. Index of adiposity
a. weight
b. length
c. triceps skinfold thickness
d. head size
92. True of growth, except
a. weight as acute indicator
b. head circumference an indicator
c. height is a poor indicator because it is
affected by fat and fluid
A 93. Dengue with circulatory failure, thready
pulse, narrow pulse pressure, hypotension, cold,
clammy
skin
a. IV
b. III
c. II
d. I
94. Patient with growth retardation, presented
with tachypnea, vomiting, GFR <50%;
what to give
a. CHON >3 mg/kg/day
b. Unrestricted carbohydrates
c. Limit fat intake
d. Will give water-soluble vitamins
95. Urine osmolarity >530, Urine Na 180,
FeNa <10%, what to do
a. give isotonic solution
b. give furosemide + mannitol
96. 8 year old child with fever and abdominal
pain with note of LLQ tenderness on PE,
BP 130/90; UA showed hematuria and (+)
WBC, what to do next
a. ASO
b. C3
c. Creatinine
d. Lower abdominal ultrasound
D
2
97. Differentiate RTA type 1 from RTA type
a.
b.
c.
d.
hypercalcemia, nephrocalcinosis
negative anion gap
A 98. Patient had VUR on VCUG with normal
UTZ, what to do next
a. prophylactic antibiotics
b.
c. urine CS then treat
d. DMSA
D 99. Most common cause of injury among
adolescents
a. suicide
b. poisoning
c.
d. motor vehicle accidents
D 100. False statements regarding sexual abuse,
except
a.
b. 5 year old
c. 6 year old can invent stories regarding
sexual abuse
d. normal PE and absence of forensic
evidence does not rule out sexual
abuse
A
101. When does human life begin
a. fertilization
b. implantation
c. when heartbeat starts
A 102. There is a need for ICU set-up because of,
except:
a. increased monitoring of vital signs
b. skilled nurses
c. increased patient:nurse ratio
A 103. Genetic counseling is accepted in the
following cases, except
a. termination of pregnancy
b. adoption
A 104. Can be diagnosed prenatally among
disorders of carbohydrate metabolism
a. Pompe’s disease
b. Tay-Sach’s disease
c. Gaucher’s disease
A
105. Autosomal dominant
a. achondroplastic dwarfism
A 106. Absence of distal femoral epiphysis in
neonates is a sign of
a. prematurity
b. hypocalcemia
c. osteogenesis imperfecta
d. hypothyroidism
A
107. Highly recommended by the DOH
a. Vitamin A and Iron
b. Vitamin C
c. Multivitamins
d. Vitamin B complex
A 108. Most prevalent vitamin deficiency seen in
70% of the population according to DOH
a. Vitamin A
b. Zinc
c. Calcium
d. Vitamin C
109. Untreated esophagitis may lead to
deficiency of this mineral
a. Zinc
b. Vitamin C
B 110. Immunization of a 14 year old prior to
transfer to a foster home
a. DPT
b. Td, MMR, Hepatitis B
c. Varicella, Hepatitis A, Hepatitis B
A
111. Drug which causes cataract
a. radiation
b. copper
c. steroids
A
112. EPI vaccines
a. BCG, OPV3, DPT3, measles, hepaB3
b. MMR
B 113. Despite short pediatric visits, it has a
lasting effect because of
a. administration of vaccines
b. continuation of care
c. counseling on nutrition
d. counseling on prevention
D
114. Physician:patient relationship is based on
a. paternalism
b.
c. physician’s skills
d. mutual trust and confidentiality
c.
d.
D 121. 39weeks, 3.6 kg with mother having
vaginal discharge 2 weeks prior to delivery
a. neonatal hypothyroidism
b. neonatal diabetes mellitus
c. neonatal hyperthyroidism
d. neonatal pneumonia
D 122. Pathologic jaundice if jaundice persists
longer than
a. 3 days
b. 7 days
c. 10 days
d. 14 days
e.
D
123. Most common cause of neonatal seizures
d. hypoxic-ischemic encephalopathy
D
124. 9 year old with nocturnal enuresis
a. environmental stressors
b. family history
c. poor school performance
d. all of the above
A
125. Treatment of central DI
a. DDAVP
C
126. Murmur of RF except
a. MR
b. MS
c. PI
d. AI
B 115. Provocative test for developmental hip
dysplasia
a. Ortolani’s maneuver
b. Barlow’s maneuver
116. Dental assessment of an 11 month old
infant without primary teeth
a. determine Vitamin D
b.
c. mandibular xray
d. all of the above
C 117. Needs antibiotic prophylaxis for infective
endocarditis
a. endotrachial intubation
b.
c. dental extraction
d. cardiopulmonary resuscitation
C 118. 4 year old with abdominal pains,
anorexia, and pallor
a.
b. peptic ulcer
c. intestinal parasitism
A 127. Child presents with diarrhea and
vomiting, prolonged CRT
a. hypovolemic shock
b. cardiogenic shock
c. neurogenic shock
d. distributive shock
C 128. GBS except
a. ascending paralysis
b. CSF: CHON ↑, low sugar, 0 cells
c. CSF: CHON ↑, normal sugar, 0 cells
d. Responds to IVIg
B 129. Most common cause of pleural effusion in
children
a. Tuberculosis
b. Congestive heart failure
c.
d. bacterial pneumonia
A
119. Management of HMD
a. oxygenation, surfactant, antibiotics
b. bicarbonate, surfactant
c. oxygenation, bicarbonate
A 120. True of transient tachypnea of the
newborn
a. slow absorption of lung fluid resulting
in increased pulmonary compliance
and tidal volume and decreased dead
space
b. resolves in 1 week
air bronchogram with reticulogranular
pattern
clear of rales and wheezes
130. Correlates with intelligence
a. adaptive
b. fine motor
c. personal social
d. gross motor
B 131. Child eats grapes and choked, presents
with intermittent wheezing
a. stop-valve effect
b. ball-valve effect
B
132. Head circumference is monitored up to
a. 1 year
b. 3 years
c.
6 years
A
133. Antidote for iron poisoning
a. deferoxamine
D
134. Full arrest in children 2º to
a. primary cardiac pathology
b. failure of resuscitation
c.
c. primary respiratory pathology
B 135. Asymptomatic non-TB mycobacterium
lymphadenitis
a. treat with INH, Rifampicin
b. complete surgical excision
c. wait for suppuration than do incision
and drainage
d. partial excision and biopsy
D 136. Absorption, distribution, metabolism,
excretion
a. synergism
b.
c. pharmacotherapeutics
d. pharmacokinetic interaction
137. 5 month old with meningitis, had neighbor
with similar case, child has recommended
vaccines. What is the etiologic agent?
a. Grp B strep
b. H. influenza
c. N. meningitides
B 138. Widespread use of this vaccine had most
effect on eradication of serious infection
a. N. meningitides
b. H. influenza
c. Typhoid
C 139. Hemorrhagic fever with renal
manifestations, rat-borne, discovered in Korea in
1993
a. Legionella
b. Leptospirosis
c. Hantavirus
A 140. Vector Culex tritaeniorhyncus
summarosus, nighttime-biting mosquito which
feeds preferentially
on large animals and birds presented with
encephalitis and died within 10 days
a. Japanese B encephalitis
b. Arbovirus
c. St. Louis encephalitis
d. Eastern equine encephalitis
PPS EXAM RECALL
NOV 10, 2004
1.
Snowman sign is seen in patients with
a. TAPVR
b. TGA
c. TOF
2.
Congenital anomalies associated with infants
of diabetic mothers
a. renal venous thrombosis
3.
Diagnosis of an infant in the nursery with
cyanosis when calm but resolves by crying
a. TEF
b. choanal atresia
4.
A case of a 4 year old female on
immunosuppresive medications contracted
varicella from a brother. What would you
give?
a. IVIG b. varicella Ig c. varicella vaccine
5.
What is the basal fluid requirement of a 3 y/o,
5 kg child?
a. 1000 mL
b. 400 mL
c 750
mL (p. 141 Nelson 16th)
6.
TB Meningitis Stage II is characterized by
a. drowsiness, stiff neck
b.cranial nerve
palsies
c. hemiplegia
7.
Case on how to distinguish between a
streptococcal from a non-streptococcal
infection?
a. antistreptolysin O > 133 todd units
b. throat swab culture
8.
Treatment for streptococcal throat infection
a. Benzathine Pen G 1.2 M U IM single dose
b. Pen V K for 10 days
9.
rash of Henoch Schonlein Purpura
d.
>5 WBC in the CSF
14. Most
common
cause
Hydrocephalus
a. aqueductal stenosis
b. post-infectious
15. A 2-year old….
a. role play b. ride tricycle
name
of
congenital
c. knows full
16. Non-billous projectile vomiting in a male
newborn
a. pyloric stenosis
17. A child diagnosed with Iron deficiency
anemia. What happens after iron therapy in 7296 hours?
a. reticulocytosis
b. increase hemoglobin
c. corrected hematocrit
18. Indications for assisted ventilation in a
newborn
a. pH < 7.25
b. pH > 7.25
c. paO2 < 40
d. pCO2 > 50
19. What will make you worry?
a. Palmar reflex at 4 months old
b. Moro reflex at 2 months
c. Parachute reflex at 7 months
d. Tonic neck reflex at 6 months
20. Treatment for Kawasaki
a. IVIG and aspirin
10. Characteristics of intestinal amoebiasis except
a. carrier state should be treated
b. infection by a 4-nucleated cyst
c. pathologic state caused by trophozoite
d. most common cause of bloody diarrhea in
the Philippines
21. What is the correct distribution of calories in a
well nourished child in terms of proteins, fats
and carbohydrates?
a. 30%-10%-60%
b. 35%-15%-55%
11. A patient with TOF who presents with on and
off headache. Hematocrit was .68. What would
be the initial management?
a. hydrate
b. request cranial CT scan
22. Characteristics of viral pneumonia on chest xray except
a. interstitial infiltrates
b. consolidation
12. Child involved in a vehicular accident who
present at the ER in shock. We insert an
intraosseous line to prevent
a. metabolic acidosis
b. hyponatremia
c. hypokalemia
13. Characteristics of Guillain Barre syndrome
except
a. areflexia/hyporeflexia
b. predominantly sensory manifestations
c. progressive muscle weakness
23. Case of a patient with cough for 2 weeks,
except
a. TB
b. pertussis
24. Which of the following
drugs needs a
maintenance dose which exceeds the MIC?
a. B lactams
b. Macrolides
c. Glycopeptides
d. Aminoglycosides
25. A boyscout went to Palawan, after 2 weeks
developed paroxysms, hepatomegaly, etc, etc,
What Plasmodium species is responsible?
a. P. malariae
b. P. vivax
c. P. ovale
d. P. falciparum
26. How would you differentiate between an
uncomplicated respiratory tract infection and
bacterial pneumonia?
a. duration of illness more than 10 days
b. color of the nasal discharge
27. A 3 y/o patient brought in because she was
allegedly underweight. She is exposed to a
Grandfather with occasional hemoptysis and
on and off intake of anti TB drugs. On PE
patient was undernourished. PPD done was
5mm induration. Chest X-ray normal. Classify
patient
a. Latent TB b. disseminated TB
c.
active TB d. drug-resistant TB
28. Treatment of choice for patient above
a. INH x 9 months
b. 2 months HRZ, 4 months HR
c. 2 months HRZS, 7 months HR+/-S
29. A male child is characterized to have
precocious puberty if he develops secondary
sexual characteristics by what age?
a. 10 ½ y/o
b. 9 ½ y/o
c 11 ½
y/o d. 12 ½ y/o
30. First sign of puberty in a male
a. deepening of the voice
b. testicular enlargement
c. elongation of the penis
31. 20/20 vision is achieved by
a. 4 years old
b. 6 years old
32. Indication for ROP screening
33. External genitalia is distinguishable by what
age?
a. 8 weeks b. 10 weeks c. 12 weeks d. 16
weeks
These are some of the questions and choices we
remembered. The incomplete choices for some of
the questions here does not necessarily contain the
correct answer, kaya read nyo pa rin yung topic.
It’s hard to remember them all since mostly cases
and they were long. It also helps to have friends
from the other hospitals baka they have a more
complete recall. At least we hope this will give you
an idea of the contents of the exam. Good luck and
God bless to all.
To Tisha (and Mark), who provided our official
review place, not to mention the food esp Mark’s
pesto and Tisha’s bangus sisig…YUM! and Girlie
(and Kiko) for our review while stuck in traffic and
for the blessed pencil during the exam,
THANKS!!! Review tayo ulit for the orals. To our
batchmates who prayed for us and gave their
support, THANKS!!!
TO GOD…THE GREATEST
GRATITUDE OF ALL.
Ge1104
PPS Exam November 2004 Recall:
1. granulation tissue- treat with silver
nitrate
2. retinoblastoma- chromosome 13
3. co A –Turner’s
4. case of myocarditis
5. causes of myocarditis - Coxsackie
6. disease that is potential to be
pandemic- a.HIV,
b.influenza,c.Hib,d.hepatitis
7. P. falciparum
8. crying peaks-___wks
9. vitamin D - rickets
10. Transient hypogammaglobulinemia
11. TOF-management
12. JRA case
13. least in the management of PSGNtransfusion with PRBC
14. Kawasaki- IVIg and aspirin
15. percentage of acquiring allergy if
father is asthmatic and patient has
sinusitis
16. fluid requirement-case
17. IVH grading
18. Asthma classification-mild, moderate,
severe – ans: moderate persistent
19. object permanence-9 months
20. non strep and strep pharyngitis - do
throat swab
21. treatment of strep pharyngitis
22. can draw a man with 2 parts- 2 y/o
23. patrhogenesis of immune mediated
thrombocytopenia
24. description of rashes-HSP
25. hemophilia
26. chromosomal aberration-MR
27. blood picture of SLE
28. Erb Duchenne
29. not needed for normal growth- lead
30. upper midarm circumference – very
good gauge of nutrition for under 6 y/o
31. GBS- not a dx criteria-predominance
of sensory impairment
32. 1st milk teeth-mandibular central
incisors
33. anal stage- 12-36 month
34. Fe therapy at 72-96hrs- what
happens- reticulocytosis
35. measles bronchopneumonia-suddenly
had cyanosis- cause is-a. ards, b.
ruptured pneumatocoele, c.
congestive heart failure, d. pyothorax
36. side effects of erythromycinnephrocalcinosis, gray baby
syndrome, enamel hypoplasia, pyloric
stenosis?
37. True of idiopathic nephritic syndrome
except: azotemia, edema, proteinuria,
hypoalbuminemia
38. stop breastfeeding - cimetidine,
nicotine, chloramphenicol
39. what is true of infants with HIVchoices unrecalled
40. what differentiates uncomplicated
URTI from chronic sinusistis: color and
character of secretions, duration of
symptoms for more than 10 days?
41. most common cause of diarrhearotavirus
42. True of amoebiasis:a. most common
cause of bloody diarrhea, b. cyst is the
infective stage,c. treat carriers of cyst,
d. pathogenic-trophozoites
43. in the treatment of OM what is not
true? a. uncomplicated-give 5-7 days
antibiotics,b.myringotomy,c. middle
ear effusion-initiate treatment with
antibiotics
44. true of candida: a. fluconazole as
DOC, b. most common cause of
diaper rash,c. decrease incidence in
the NICU due to improved
conditions,d. non candica albicans sp.
as the cause
45. patient on prednisone for nephritic
syndrome exposed to varicella:a. give
IVIg, b. give VZIg’c. decrease
prednisone
46. testicular enlargement earliest –a. 7
½, b. 9 ½ c.10 ½, d. 11 ½
47. first sign of puberty in females- brest
bud
48. pyloric stenosis- hypochloremic
metabolic alkalosis
49. True of NEC-a. e of onset inversely
proportional to appearance of
symptoms, b. pneumatosis intestinalis,
c. complicated if located at the ileum
50. NB neural tube defect-a. low set ear,
B. alcoholism, c. hypothyroidism, d.
methimazole
51. Denver Developmental Screening
Test (DDST) – device to assess the
dev
Status of children during the first 6 yrs
of age
52. Weight / height / HC computations
53. What is a toxoid ?
54. Breast Milk vs Cow’s Milk –
composition / advantages
55. Trisomy 21
56. AD, AR, X-linked diseases
57. Klinefelter – tall
58. Caput vs cephalhematoma
59. PTB - latent
60. Toxocariasis – lung mxs
61. Rash of chicken pox – describe :
cicatricial
62. Snowman – TAPVR
63. Sinuses – ethmoid, frontal – which is
seen first ?
64. Kerosene – Hydrocarbon ingestion –
case – mgt?
65. Case diarrhea – mgt?
66. Intussusception case
67. Choledochal cyst – case
68. Dx of UTI – KUB done wat next ?
VCUG
69. Mitral stenosis – RHD
70. umbilical hernia 0.5-1 cm- observe?
71. Neonatal seizure
72. Absence or petit mal – daydreaming,
poor school performance
73. Osteosarcoma vs ewings
74. Galactosemia – (+) non glucose
reducing substance in urine
75. Acromegaly vs Gigantism case
76. CAH – 21 hydroxylase deficiency
77. Folic acid
78. ITP
79. what to request: px
hypotonic,protruding tongue, jaundice
– ans: thyroid function test
80. what to do for a patient who had
bitten an electric cord, lips were
burned- admit?
81. ethics of referrals
82. CMV question
83. Presents with eosinophilia 90% of the
time: a. toxocara canis, b.
ancylostoma, c. trichura, d. enterobius
84. x chromosome, MR- Klinefelter
85. disruptive behavior common in:
a.cross country adoption,
b.abandoned child, c. multiple foster
care
86. least medical complication of obesity:
hypertension, cholelithiasis, late
menarche, MI
87. what is true with the management of
burns- remove clothes, immerse in tub
of water
88. optimal asthma managementmonitoring lung function
89. patient with crt 5 sec, with diarrhea, no
signs of dehydration, tolerates ORS,
what to do?a. send home, b. bring to a
room with ok temp, c. give O2, d.
hydrate
90. best or true pulse ox: a. BPincreased,b. adequate perfusion,c. ok
temp, d. all
91. endorser of products, infraction of
responsibility?
92. Baby with neural tube defect and low
set ears, etiology- a.Syphilitic mom,
b. DM, c. alcoholism, d.
hyperthyroidism
93. cerebral calcification, jaundice: a.
toxoplasmosis, b. CMV
94. differentiate AML from ALL:
leucocytosis
95. ALL-most common site of relapseCNS
96. What is true of PDA
97. x linked recessive: color blindness,
hemophilia
98. when will a positive ppd appear with
mycobacterial infection
99. Tx for (+) exposure, 5 cm PPD- 9 mos
INH
100 .needs ventilatory support: pH <7.2,
pCO2= 50, pH> 7.4, pO2 40 and less
101. True of bresastmilk except:
a.breastmilk will provide vit k for
protection against hdn, b.something
about IgA, c.more will have allergy
with breastmilk
102.stimulates complement fixation: IgA,
IgG,IgM, cytokine
103.variability in response to vaccination is
due to: chemical stability, is genetically
predetermined, age, immaturity of the
immune system
104.vaccines-postnatal age
105. When is transfusion of PRBC
warranted: a. NB PTon CPAP, FiO2 21%,
Hct
40; b. term asymptomatic, Hct 28;
c. term, with pneumonia Hct 35; d. PT
gaining weight Hct 32
106.abscess after I and d, what is not
effective: Cloxacillin, cefalexin, amoxicillin,
cotrimoxazole
107.Management of OM: myringotomy,
give…
108.boy VA in shock,, gave intraosseus
LRS and NSS to prevent a. acidosis,
b. hyponatremia, c. hypokalemia
109.normal neuro exam, 3 y/o, (+)
seizures, oliguric, BP 120/80: nephritic
syndrome, TB meningitis, PSGN
110.not a frequent cause of sinusitis:
staphylococcus, streptococcus,
H. influenza, moraxella
111.cause of prolonged cough: M. tb,
Bordetella, Strep pneu
112.atypical pneumonia: mycoplasma,
legionella, H. influenza
113.Zollinger Ellison-gastritis
114.Gastritis-Helicobacter pylori
115.Bilateral choanal atresia- cyanotic
when mouth is closed, pink with crying
116.true B. thalassemia minor-needs Fe,
hypochromic anemia?
117.Blood culture-(+) urinalysis- infective
endocarditis
118. expected response when a patient is
given a vaccine
119. rabies-(-) negri bodies safely rules out
rabies?
120.True of cryptococcus-a.meningitis
subacute and chronic;b.frombirds and
pigeons,c. lab tech at greatest risk
121.least risk to develop military tb - a. 4
y/o , b.adolescent,
c.immunocompromised, d, with other
medical conditions?
122.NB with ground glass abdomen:
meconium ileus, peritonitis, malrotation?
123.3 wks old with jaundice, acholic, with
hepatomegaly,dx?-a.inspissated bile
syndrome, b.biliary atresia, c.choledochal
cyst,d.hepatitis
124.20/20 vision –4y/o
125.(+) HBs Ag- 1st dose vaccination
before discharge, 1st dose vaccination at
2 mos?
126.Post Strep GN - a.low C3, b.(-) Ds
DNA or ANA, c. Hgb <9, d.ASO titer
127. pancreatitis
128. hydrocarbon ingestion- what would
you do if temp is 40C and lungs clear:
a. CXR, b. Gastric Lavage, c.Give
antibiotics
129. 12 y/o, (=) fever, (=) arthritis for 3
wks: diagnosis: a. JRA, b. post
infectious arthritis, c. SLE, d.
ankylosing spondylitis
130. will have generalized headache
except: a. hypertension, b. sinusitis,
c. fever, d. hypoglycemia
131. congenital hydrocephalus - arnold
chiari, aqueductal stenosis, post
infectious, dandy walker?
132. neonatal seizures, true of, except: in
SGA after 2 wks, generalized tonic
clonic?
133. not true : moro present at 2, palmar
at 4 wks(2-3 wks), tonic neck reflex
at 6 wks, parachute at 7 mos
134. diagnostic criteria of TSS except:
thrombocytopenia, fever 38C, rash
135. complications of steroid use:
psychoses, cataract and glaucoma,
hypertension, bone demineralization?
136. pancytopenia
137. predispose to cancer in childhood:
trisomy 18, burkitt’s lymphoma,
papillomaviruses
138. bilateral clubfoot: conservative
management with splint, serial
casting
139. 6 toes, not aligned well articulated,
what to do: observe, no harm;
surgery at 1 y/o
***this is just to give you an idea of the
coverage, not so sure of the answers,
questions are rephrased the way we
understood them. GOOD Luck and GOD
BLESS!!
Recalled Exam Questions - PPS
NOVEMBER 15, 2006
1. The basic pathology in rheumatic diseases
is
a. always triggered by an
infectious process
b. “self-tolerance” to own’s
tissue is lost
c. ?
d. ?
Ans: B Nelsons pp 793
2. Pica is associated with which of the
following:
a. Toxocariasis
b. Strongyloidosis
c. Ascariasis
d. -----------Ans: A Nelsons pp 1165. Classic
presentation of VLM includes
eosinophilia, fever, hepatomegaly
and occurs most commonly in
toddlers with a hx of pica and
exposure to puppies
3. Description of Rumination disorder
(Nelsons pp 73) Hallmark of this
disorder is weight loss or failure to
gain weight because of repeated
regurgitation of food without
nausea or associated GI illness
4. A mother with unknown hepatitis status
gave birth to a baby girl, you should
a. Give the baby hepatitis B
vaccine only
b. Give the baby hepatitis B
vaccine and Ig
c. Give the baby hepatitis B
vaccine and draw blood to
determine HbsAg status of
baby
5. The triad of Osteogenesis imperfecta: (
always asked ito)
Triad: fragile bones, blue sclerae and
early deafness. Nelsons pp 2336
6. The best diagnostic test for Peptic Ulcer
Disease in children
a. endoscopy
b. biopsy for H. pylori
c. abdominal x-ray
d. urea breath test
ans: A Nelsons pp 1245 endoscopy is the
method of choice to diagnose PUD in children
7. Drugs contraindicated for G6PD babies
* nelsons pp 1637
8. PDA is commonly associated with which
of the following :
a. VSD
b. ASD
c. Pulmo-aortic window
d. Coarctation of the Aorta
Ans: D (?) Nelsons pp 1511
9. Patient with unilateral nasal discharge,
noisy breathing and episodes of cyanosis
a. Choanal atresia
b. Severe nasal septum deviation
c. Foreign body
d. Tracheo-esophageal atresia
e.
10. Most hydroceles resolved by
a. 1 years old
b. 2 years old
c. 3 years old
d. 4 years old
Ans: A. Nelsons pp 1820. Most
hydroceles resolves by 12 months.
Hydroceles that persist beyond 1218 months may require surgical
correction.
11. Which of the following best describes
appendicitis
a. ?
b. ?
c. Perforation is more common
in adolescents
d. Highest incidence in early
childhood
12. Raynaud’s phenomenon is commonly
associated with
a. SLE
b. JRA
c. PAN
d. Scleroderma
ANS: D Nelsons pp 817
Raynaud’s phenomenon results from digital
arterial spasm is often the earliest
manifestation of Scleroderma, induced by
exposure to cold and has 3 stages : pallor,
cyanosis and erythema
13. Indications for tonsillectomy and
adenoidectomy except
a. 2 episodes of tonsillitis the
previous years
b. Tonsillitis unresponsive to
antibiotics
c. Obstructive symptoms or
snoring which disturbs sleep
with failure to thrive
d. Mother wants to enroll child
at a pre-school
ANS: D Nelsons pp 1397
Criteria for tonsillectomy accdg to
Childrens Pittsburgh study: 7 or
more throat infections tx with
antibiotics in the preceeding year ,
5 or more infections treated in
each of the preceeding 2 year or 3
or more infections treated with
antibiotics in each of the
preceeding 3 year.
14. Drug of choice for MRSA
a. Vancomycin
b. Penicillin
c. Oxacillin or nafcillin
d. ( I forgot…..dolan kaya?)
hehe
Ans: A
d. Suzuki…..Honda…Hyundai
15. Fever pattern which occurs every 3 days (
this question was very vague….)
a. Dengue
b. African hemorrhagic fever
c. P.vivax
d. Leptospirosis
16. Pancytopenia without organomegaly
a. ALL
b. CML
c. AML
d. Aplastic anemia
17. Steve Irwin ( zino zya??? Secret…dehins
cya kapatid ni Zsazsa zaturnah mga
ineng…) was attacked by a sting ray
causing his death. The mechanism of his
fatal death is
a. Urticaria
b. Heart block
c. severe infection
d. Anaphylaxis
Ans: B. Steve Irwin, the
crodocodile hunter of Australia
was attacked by a stingray
piercing his heart. (moral lesson:
manood ng discovery channel
paminsan2x…hwag PBB lagi or
telenovela…take note Minnie and
arjie…hehehe)
18. Most common cause of death among
toddlers and pre-schoolers next to falls is
a. Motor/vehicular accidents
b. Submersion/drowning
c. Poisoning/ingestion
d. Sports injuries
ANS: B? Nelsons pp 257
19. Best diagnosis to delineate kidney
anatomy and renal mass or tumors
a. CT scan
b. Pelvic Ultrasound
c. Xray
d. VCUG
20. A completely immunized child according
to EPI
a. DPT OPV BCG MMR
b. BCG DPT OPVE Measels
Hepa
c. BCG DPT OPV Varicella Hib
21. Fever with that exhibits a stepladder
pattern
a. Typhoid
b. Kawasaki
c. Dengue
22. Best decribes FUO
a. Fever for more than 1 week
b. Fever documented by
caretaker’s history and
validated by mercuric
thermometer (?)
c. ?
d. ?
FUO Nelsons pp 843
23. A 10 year old boy regularly plays soccer
however lately would usually complain of
DOB and chest tightness lasting 30
minutes
a. Evaluate for asthma
b. Cardiac work-up
c. ?
d. ?
Ans: A
24. Classification of asthma (Please memorize
table on classification and management
GINA handbook)
25. Which of the following is least helpful in the
diagnosis of Retinoblastoma before enucleation (?)
a. CT-scan
b. UTZ
c. Biopsy
d. Direct examination
Answer: Biopsy ? answer ni Bong
26. Features of Infantile glaucoma EXCEPT
a. Epiphora ( tearing)
b. decreased visual acuity
c. eye squeezing
(blepharospasm)
d. photophobia
ans: B. Nelsons pp2122. classic
triad of infantile glaucoma is
photophobia, blepharospasm and
epiphora. Visual acuity can not be
illicited in infants.(oo nga ano?)
27. 6 year old child who tilts her head when
watching TV, decreases her palpebral
fissure and missed some letters on the
snellen chart
a. Myopia
b. Hypertropia ( not
HYPEROPIA)
c. Astigmatism
d. Glaucoma
Answer: B …sabi ulit ni Bong.
Nelson’s pp 2094. Hypertropia is
a form of paralytic strabismus, 4th
nerve palsy . Child exhibits head
tilt to the shoulder opposite to the
affected eye. Amblyopia is
uncommon. Head tilting is not
present in myopia, astigmatism
and glaucoma.
28. General pediatrician refers a patient to a
pedia-pulmo. On follow-up, patient’s
mother decides to follow-up with the
specialist. If you were the pulmonologist
what would you do
a. Refer patient back to the GP (
kung friend mo cya…hehehe)
b. Accept the patient as yours
since it’s the mother’s choice
c. ?
d. ?
 there are some questions on
professional ethics …
29. Best describes prognosis of untreated
classic MSUD
a. Severe MR and death with in
5 years
b. Severe MR and death with in
1 year
c. Death with in a few months
d. Death with in a week
Ans: D Nelsons pp 409
30. Normocytic, normochromic anemia
a. Thalassemia
b. Anemia of chronic diseases
c. Hemolytic anemia
d. Aplastic anemia ( im not sure
of this )
31. Forscheimer spots are found in
a. Rubella
b. Rubeola
c. Mumps
d. Diphtheria
Ans: A Nelsons pp 1033.
Forscheimers spots are discrete
rose colored spots on the soft
palate that may coalesce into a
red blush and extend over the
fauces
32. Micronutrients which causes mental
retardation if deficient in intake
a. Zinc
b. Iodine
c. ?( poor memory na talaga)
d. ? ( m sure di ako nabigayan ng
nanay ko…)
Ans: Iodine
33. DOC for candidemia
a. Nystatin
b. Amphotericin
c. Ketoconazole
d. Fluconazole
ANS: B Nelsons pp 1012
34. Interstitial lymphocytic pneumonia is
seen in
a. HIV
b. Cant recall other choices
* this is the only question on AIDS and
this question was also asked during previous
exams
35. Newborn who manifests bilious vomiting
and on abdominal x-ray shows ground
glass appearance
a. Meconium ileus
b. Volvulus
c. Malrotation
d. Intussusception
ANS A. Nelsons pp 1234
36. Pharmacodynamics is best defined as
a. Labeling of drug
b. Manufacturing of drugs
c. Drug absorption, excretion,
metabolism,distribution (
pharmacokinetics)
d. Relationship of pharmacologic
response to drug
concentration in blood.
Ans: D . Nelsons pp 2427
37. Polycythemia
a. Increase RBC Increase blood
volume
b. Increase RBC, decrease blood
volume
c. Increase RBC, normal blood
volume
ans: A . Nelsons pp1641. True
polycythemia is characterized by increase
of both the RBCs and the total blood
volume
38. Physiologic anemia of infancy begins at
a. ?
b. ?
c. 8-12 weeks
d. 6 weeks
Ans: C . nelsons pp1610
39. The ff denotes poor prognosis in
meningococcemia patients
a. Aseptic meningitis
b. Hyperpyrexia
c. Purpura appearing with in 12
hours after onset of fever
d. Recrudescence of fever
Ans: D Nelsons pp 899
40. Newborn after 3 days developed
respiratory distress. Chest x-ray shows
pulmonary edema and cardiomegaly, distal
heart sounds with tachycardia
a. Sepsis
b. HMD
c. Myocarditis
d. Pneumonia
41. Features of congenital varicella
42. Compute for weight of a 9 month old
infant with a birthweight of 3.3 kgs
Mnemonics: wt in gms = age in mos x
600 + BW
Ans: 9.6 kgs ( this question repeated
verbatim…hehehe)
43. A child with a height of 100 cms is
approximately
a. 4 yr old
b. 5 yr old
c. 6 yr old
d. 7 yr old
Ans: A
mnemonic for height: age in years
x 5 + 80
44. A urine collected thru a specimen bag has
a false positive rate of
a. 60%
b. 55%
c. 70%
d. 85%
Ans: D. CPG Approach to UTI pp 8
45. Renal agenesis is seen in
a. Oligohydramnios
b. Polyhydramnios
c. ?
d. ?
Ans: A .(Nelsons pp 1783 Renal
agenesis)
46. ABG’s ph 7.4 Bicarbonate 23 PO2 60
PCO2 40 sats 95%. The present mech
vent settings are the ff: Fi02 80 RR 40 PIP
15 PEEP 5. What should you do:
a. decrease RR
b. Increase PIP to 20
c. Increase PEEP
d. Increase FiO2
e. Call Dr Herbie or Dra A.E.
Ans: A? for ABG’s /mech vent, I
referred to Gomella
47. 5 month old with a liver span of 5 cm,
palpable edge 2 cm below costal margin
a. Order abdominal scan
b. Refer to a hepatologist
c. Follow-up after 1 year
d. Order liver enzymes
ans: c Nelsons pp 1309
48. The mildest but most chronic form of
malaria
a.falciparum
b.Vivax
c.malariae
d.ovale
ANS: C. Nelsons pp 1140
49. Question on NEC
50. Follow-up of a newborn who was
discharged less than 24 hours old
a. With in 48 hours
b. With in 3 days
c. With in a week
d. With in 2 weeks
Ans: A ( pp 1 of Preventive
Pediatric Handbook)
51. Which one the following is a red flag?
a. Sits unsupported at 7 months
b. Hands still fisted at 4 months
old
c. Unable to hold objects at 5
motnhs old
d. Unable to walk at 10 months
old
Ans: B ( Preventive Pediatric
Handbook)
52. Most common error in blood pressure
taking
a. deflating the cuff very rapidly
b. innapropriate size of the cuff
c. failing to accurately note 1st
korotkoff sound
d. ?
Ans: A/B . Nelsons pp 1482
53. Least priority in evaluating for FUO
a. CBC
b. CXR and PPD
c. Torniquet test, HCT and
platelet
d. Blood and Bone marrow
culture
Ans: c? FUO dxtics pp 846
54. Nagayama spots are a feature of
a. Rubella
b. Rubeola
c. Schistosomiasis
d. HHV 6
Ans: D ( a.k.a. Roseola Nelsons pp
1070. Be careful sometimes they use
“synonyms”)
55. Clinical effects upon withdrawal of drug
responsible for drug fever is seen with in
a. 24 hours
b. 48 hours
c. 72 hours
d. 96 hours
Ans: C . Nelsons pp 844…
Discontinuation of the drug is
associated with resolution of the
fever generally with in 72 hrs
although certain drugs such as
iodides are excreted for a
prolonged period with fever that
may persist for as long as 1 month
56. 4 year old male with chief complaint of
sore throat, PE revealed erythematous
ulcers on mouth and gums
a. Viral Stomatitis
b. Herpetic
c. Vincent’s angina (?)
57. Best describes pseudomembrane of
diphtheria
a.
Leatherlike and
adherent
b.
Erythematous and
bleeding
c.
?
d.
?
Ans: A Nelsons pp 887
58. Post exposure treatment for Pertussis
a. Erythromycin
b. Rifampicin
c. Ciprofloxacin
d. Co-amoxiclav
e.
Ans: A pp 889
59. Describes Habit cough except
a. Disappears during sleep
b. Resistant to anti-cough
medications
c. Associated with voice
hoarseness
d. Disappears once examining
MD goes out of the room
Ans: C Nelsons pp 1403
60. Sinuses fully developed at 7 years old
a. Frontal
b. Ethmoidal
c. Maxillary
d. Sphenoidal
Ans: A
61. NEC
a. Age of onset is inversely
proportional to AOG
b. most common site of
perforation is distal ileum
c. most common sx is abdominal
distention
62. Newborn home-delivered, breastfeeding
purely with 1 episode of melena,
unremarkable maternal hx. DX?
a. aspiration of maternal blood
b. NEC
c. Vitamin k deficiency
d. ?
Ans. C . nelsons PP 1668. we
think this is a case Post-neonatal
Vitamin k def
63. A female school-age child exhibits blank
stares and dropping of objects
a. petit mal seizures
b. rolandic seizures
c. juvenile myoclonic epilepsy
d. ?
Ans: A (absence or petit mal
seizures) Nelsons pp 1997
64. Newborn presents with the ff blood picture
( cant recall, can only remember the
choices sori!)
a. ALL
b. CML
c. Congenital ALL
d. Congenital AML
Note: familiarize na lang smear results of
each entity
65. 12 month old infant still without teeth
a. work-up for thyroid problem
b. assure parents of familial
trend
c. wait until 16 months old
d. bigyan ng toothbrush hehe
Ans: Sa nelsons acceptable up to
13 months while sa del Mundo up
to 16 months
66. Best diagnosis for Rheumatoid arthritis
a. trial of aspirin therapy
b. CS ( of synovial fluid?)
c. ?
d. ?
Ans: A
67. Which of the ff is not a feature of Rh and
ABO?
a. reticulocytosis and anemia
b. reticulocytopenia and anemia
68. Infant with (+) Nikolsky sign with crusting
and rashes( can not remember if there is
erythema mentioned)
a. SSS
b. TEN
c. TSS
d. Necrotizing fasciitis
Ans: A /B?Nelsons pp2225 and
pp 2183. Both SSS and TEN
exhibit nikolsky sign (denudation
of skin upon tangential pressure)
however in TEN, it is only present
in areas of erythema wheras in
SSS, it may be present even in
non-erythematous crusted areas
69. Question on mechanisms of
Immunizations/ Immunoglobulins (?)
(e.g. definitions of active, passive,
artificial immunizations)
70. Which of the following is true about fever
a. the higher the temperature the
more bacterial in origin
b. all fevers irregardless of
character warrants complete
laboratory work-up
c. always warrants antibiotics
d. a single episode may be
benign in origin and may not
warrant further investigation
but just close observation and
follow-up( or something to
this effect)
71. Situation: Mom brings a child to you with
hx of 3 days of productive cough, She
wants you to give antibiotics.What should
you advise?
72. MC site of skeletal TB
a. spine
b. knees
c. hips
d. clavicles
ans: A (TB consensus)
73. MC form of extra-pulmonary TB in
children
a. scrofuloderma
b. potts
c. renal
ans: A (TB consensus)
74. Child on HRZ develops jaundice after 2
weeks. You should:
a. discontinue Rifampicin
b. continue rifampicin but on a
lower dose
c.
75. Anti-koch’s medications which causes flulike symptoms like fever with chills
a. ethambutol
b. rifampicin
c. INH
d. Pyrazinamide
Ans: A
76. Causes Drug-induced renal tubulonephritis
a. quinolones
b. rifampicin
c. streptomycin
77. question on DOTS
78. Pregnant woman being tx for PTB, her OB
discontinues her meds. Upon giving birth,
you intervention for the newborn?
79. Diagnostic Classification of TB
80. What is common among the the ff:
dehydration, DI, atropine , anhydrotic
eczema?
a. bradycardia even if with fever
b. lack of shivering mechanism
c. ?
d. ?
Ans: B???
A. B Nelsons pp 1318
81. Mechanism of physiologic weight loss in
newborns
a. loss of ECF
b. (cant recall other choices)
82. Definitive dx of JRA
a. (+) Rf
b. ASO
c. ESR
d. CRP
Ans: A
83. Question on Rheumatic Fever
84. Neonate presents with cyanosis,
retractions with tachypnea
a. intubate and hook to mech
vent
b. ambubagging 100% O2 via
mask
c. O2 and suctioning of
secretions
Ans: A? Review NALS
85. Least consideration in rehydration phase
of AGE
a. fluids
b. anion gap
c. hyperglycemia
d. Sodium replacement
86. Considered as “secondary
chemotherapy”(?) for PTB
a. INH
b. Rifampicin
c. pyrazinamide
d. inh and rifampicin
87. Differentiate breastfeeding from
breastmilk jaundice(….walang
kamatayang jaundice)
88. I think 3 or 4 questions on the
characteristics of IG’s – IgG, IgE, IgA. Ig
M
-which Ig (G) confers maternally
derived immunity
-which Ig (E) is related to allergies
-which Ig ( A) is present in saliva
/GI secretions
-which Ig (M) reflects acute
infection
89. Golden period of doing Kasai procedure
a. 6
b. 8
c. 16
d. 24
90. Least priority in Kwashiorkor
a. anemia
b. diarrhea
c. infection
d. dehydration
91. SMR – please memorize! Always asked
92. Earliest sign of puberty in males 93. Earliest sign of puberty in females 94. Reflexes present in a 1 month old except
a. rooting
b. parachute
c. moro
d. asymmetric tonic neck
Ans: D
95. DOC for uncomplicated non-typhoidal
salmonella – Nelsons mentions 2 drugs,
ampi and cotri (pp 915)
96. Components of cow’s milk vs breastmilk
(Del Mundo)
97. Question on infectious agents causing
neonatal pneumonia
98.
99. Best describes Turner syndrome
a. Buccal smear establishes dx
b. abnormality in translocation
of chromosome
c. commonly presents with short
stature and amenorrhea ( or
delayed puberty?)
Ans: C. Nelsons pp 1931.
Mechanism is absence of 2nd X
chromosome with or without
mosaicism( not translocation).
Cardinal findings are short stature
and failure of sexual maturation.
Diagnosis is thru chromosomal
analysis.
100.
Most dramatic event for girls ages 813
a. menarche
b. breast buds
c. pubic hair
d. growth spurt
101. Most dramatic event for boys (middle
childhood ages 10-14)
e. penile enlargement
f. lymphoid tissue maturation
g. midface enlargement
h. voice changes
102. A female 16 year old from Ilocos Sur
region with vomiting and abdominal pain,
probable cause
a. capillariasis
b. filariasis
c. schistosomiasis
d. ?
Ans: A ( Infectious Handbook, yung
local Redbook natin,hehe)
103. Paralytic shellfish poisoning
a. Saxitoxin
b. ?
c. ?
d. ?
Ans: A ( this question is always asked
. Nelsons pp 2377.)
104. Minamata disease
a. mercury poisoning
105. Child personality type that is prone to
accidents
a. slow to warm kids
b. hyperactive and impulsive
c. introvert and pessimistic
ans: B
106. Rocker-bottom feet, micrognathia, cleft
lip
a. trisomy 13
b. cri-du-chat
c. trisomy 18
d. trisomy 21
ans: A (Nelsons pp 384, Table 70.1)
107. Limb abnormalities, microcephaly,
cicatrical skin lesions, expected intrauterine infection is :
a. varicella
b. rubella
c. influenzae
d. rubeola
ans: A
Hi mga lola….! Hope this will help you.
Sorry just finished encoding it…medyo tinamad
ako hehe. Please read the compilation of past
exams, it is really a great help because a lot of
questions were either repeated verbatim or
rephrased. Of course Nelsons pa rin and a little of
del Mundo ( Nutrition, G and D and Toxicology).
But please also memorize by heart the TB
consensus, the Handbook on Preventive Pediatrics
and the CPG’s. I think mga 10 -15 questions on
PTB. Only 3 questions on Cardio, mga 7 on hema,
all peripheral smear pics, grabe. As advised by
madr chin and dang, read 1st the infectious chapter
at later na G&D kasi memorization talaga mga
milestones/anthropometric calculations. We tried to
research the answers , verify nyo na rin to be sure.
The exam is 150 items to be answered in 3
hours but the 1st 30 minutes is spent on filling up
waiver forms and announcements/instructions.
Please bring a valid ID. Our exam results were
released via internet (mga midnight na ata yun or 1
am…nakakatense mag antay results!..angela tnx for
everything!) .
There’s no other way to pass except
prayers and of course studying. Pray pray and pray
harder and thank HIM afterwards..!
Miss u all guys….pay me a visit dito
Bukidnon at magcelebrate tayo…syempre kayo ang
taya….miss
ko
na
ang
St
luke’s...dorm..canteen..nsy
pantry
.BK..Morato…Gateway and of course ang
168….hehehe….Anji and me will
pray for
everybody! Motto nyo: Philippians 4:13 I can do
all things thru Christ who gives me strength…!
Promise ni Lord yan and He never fails us...can’t
wait to celebrate with you mga lola! Love
you…mwahh!!!!
Its me,
Madr Brends
108. The mechanism that explains why
females are more predisposed to
having immunologic diseases
109. Triad of Wiskott-Aldrich syndrome (
thrombocytopeni, recurrent infections
and eczema.) Nelsons pp 699
110. Most common site of TB in the CNS
a. Cerebellum
b. Cerebrum
c. Brainstem
d. Ventricles
Ans: D
111. Snowman’s sign is seen in
a. TAPVR
b. TGA
c. PDA
d. CoA
Ans: A